Courses
Courses for Kids
Free study material
Offline Centres
More
Store Icon
Store

NCERT Solutions for Class 11 Maths Chapter 1 - Sets

ffImage
Last updated date: 25th Apr 2024
Total views: 783.3k
Views today: 19.83k
MVSAT offline centres Dec 2023

NCERT Class 11 Maths Chapter 1: Complete Resource for Sets

Sets Class 11 NCERT PDF material comes handy for all the students of Class 11. The solution material can be essential for last-minute revision. Download it and get accustomed to the critical techniques to solve questions related to Sets. The best part is that you can avail it free of cost. Every learner needs to get familiar with all the concepts of the chapter as it will help in studying other Maths topics like Probability, Sequences and Geometry. Although this is relatively an easy chapter, you still need to devote a considerable amount of time and practice for the same. Set Class 11 NCERT Solutions PDF by Vedantu is prepared for your assistance.

Class:

NCERT Solutions for Class 11

Subject:

Class 11 Maths

Chapter Name:

Chapter 1 - Sets

Content-Type:

Text, Videos, Images and PDF Format

Academic Year:

2024-25

Medium:

English and Hindi

Available Materials:

  • Chapter Wise

  • Exercise Wise

Other Materials

  • Important Questions

  • Revision Notes

Points To Remember Before Solving Exercises:

1. Set: A set is called a well-defined collection of objects.

2. Representation of Sets: 

There are 2 methods to represent a set

  1. Roster or Tabular Form: In this form, we list all the members of the set within braces { } and separate by commas.

  2. Set-builder Form: In the set-builder form, we list the property or properties satisfied by all the elements of the sets.

3. Types of Sets 

  • Empty Sets: A set which will not have any element is known as an empty set or the void set or null set. It is denoted by Φ or {}.

  • Singleton Set: A set which has a single element, is known as a singleton set.

  • Finite and Infinite Set: A set which has a finite number of elements, is called a finite set, if not the set is known as an infinite set.

  • Equal Sets: Two sets A & B are said to be equal, if all elements of A is also an element of B, i.e. two equal sets will have the same elements.

  • Equivalent Sets: Two finite sets A & B are said to be equal if number of elements in both sets are equal, i.e. n(A) = n(B)

4. Subsets: Set A will be a subset of Set B if every element of Set A is also present in Set B. In simple words, set A is contained inside Set B.

For Example: If set A has {A, B} and set B has {A, B, C}, then A will be the subset of B since elements of A are also present in set B.

5. Power Set: The collection of all of the subsets of a set A is known as the power set of A. Generally this is denoted by P(A). If the no of elements in A i.e. n(A) = n, then the no of elements in P(A) = 2n.

6. Universal Set: A set which contains all sets in a given context is defined as the universal set.

7. Venn-Diagrams: Venn diagrams are diagrams that show the relationship between two sets. The universal set U is represented by a point inside a rectangle, and its subsets are represented by points within closed curves within the rectangle in Venn diagrams.

8. Union of Sets: The union of two sets A & B is denoted by A ∪ B will be the set of all those elements which are either in set A or in set B or in both A and B.

9. Intersection of Sets: The intersection of two sets A & B is denoted by A ∩ B, and it is the set of all elements which are common in both set A and set B.




Sets Chapter at a Glance - Class 11 NCERT Solutions

  • A set is a well-defined collection of objects.

  • A set which does not contain any element is called empty set

  • A set which consists of a definite number of elements is called finite set Otherwise, the set is called infinite set

  • Two sets $A$ and $B$ are said to be equal if they have exactly the same elements

  • A set $A$ is said to be a subset of a set $B$, if every element ofA is also an element ofB. Intervals are subsets of $R$

  • A power set of a set $A$ is a collection of all subsets of $A$. It is denoted by $P(A)$.

  • The union of two sets $A$ and $B$ is the set of all those elements which are either $A$ or in $B$.

  • The intersection of two sets $A$ and $B$ is the set of all elements which are common.

  • The difference between two sets $A$ and $B$ in this order is the set of elements which belong to $A$ but not to $B$.

  • The complement of a subset $A$ of universal setU is the set of all elements of U which are not the elements of A

  • For any two sets $A$ and $B,(A \cup B)^{\prime}=A \cap B$ and $(A \cap B)^{\prime}=A \cup B$

  • If $A$ and $B$ are finite sets such that $A \cap B=\phi$ then $n(A \cup B)=n(A)+n(B)$ If $A \cap B \neq \phi$, then

Competitive Exams after 12th Science

Mastering Class 11 Maths Chapter 1: Sets - Question And Answer, MCQs, and Tips for Success

NCERT Solutions for Class 11 Maths Chapter 1 "Sets" includes six exercises and a miscellaneous exercise. Here's a brief summary of each exercise:

Exercise 1.1: This exercise contains nine questions related to the basics of sets, including the definition of a set, the roster method, set-builder form, empty set, finite and infinite sets, equal sets, subsets, and and the power set of a set.

Exercise 1.2: This exercise has seven questions based on set operations, such as the union, intersection, and complement of sets, and the properties of these operations, including the distributive law, commutative law, and associative law.

Exercise 1.3: This exercise covers Venn diagrams and their use in representing sets and solving problems related to sets, including questions on the union and intersection of sets, complement of sets, and solving word problems using Venn diagrams.

Exercise 1.4: This exercise includes eight questions related to the applications of sets in various fields, such as representing data using sets and sets in computer science, and includes questions related to the implementation of sets in programming.

Exercise 1.5: This exercise contains four questions based on the Cartesian product of sets, which is used to represent the ordered pairs of two sets, and applications of Cartesian products in mathematics.

Exercise 1.6: This exercise has four questions that are miscellaneous in nature, related to set theory, and applications of sets in daily life.

Miscellaneous Exercise: This exercise contains ten questions of varying difficulty levels that cover a wide range of topics related to sets, including set theory, real-life applications of sets, and the properties of sets.


The exercises in NCERT Solutions for Class 11 Maths Chapter 1 "Sets" are designed to help students build a strong foundation in the basics of sets and their applications in different fields. The solutions provided for each exercise include detailed explanations and step-by-step solutions to help students understand the concepts better.


Practice and Master the Concepts of Sets with NCERT Solutions Exercises for Class 11 Maths Chapter 1

Exercise (1.1)

1. Which of the following are sets? Justify your answer.

i. The collection of all months of a year beginning with the letter J.

Ans-

To determine if the given statement is a set

A set is a collection of well-defined objects.

We can definitely identify the collection of months beginning with a letter J.

Thus, the collection of all months of a year beginning with the letter J is the set.

ii. The collection of ten most talented writers of India

Ans-

To determine if the given statement is a set

A set is a collection of well-defined objects.

The criteria for identifying the collection of the ten most talented writers of India may vary from person to person. So it is not a well-defined object.

Thus, the collection of the ten most talented writers of India is not a set.

iii. A team of eleven best cricket batsmen in the world.

Ans-

To determine if the given statement is a set

A set is a collection of well-defined objects.

The criteria for determining the eleven best cricket batsmen may vary from person to person. So it is not a well-defined object.

Thus, a team of eleven best cricket batsmen in the world is not a set.

iv. The collection of all boys in your class.

Ans-

To determine if the given statement is a set

A set is a collection of well-defined objects.

We can definitely identify the boys who are all studying in the class. So it is a well-defined object.

Thus, the collection of all boys in your class is a set.

v. The collection of all-natural numbers is less than $100$.

Ans-

To determine if the given statement is a set

A set is a collection of well-defined objects.

We can identify the natural numbers less than $100$ that can easily be identified. So it is a well-defined object.

Thus, the collection of all-natural numbers less than $100$ is a set.

vi. A collection of novels written by the writer Munshi Prem Chand.

Ans-

To determine if the given statement is a set

A set is a collection of well-defined objects.

We can identify the books that belong to the writer Munshi Prem Chand. So it is a well-defined object.

Thus, a collection of novels written by the writer Munshi Prem Chand is a set.

vii. The collection of all even integers.

Ans-

To determine if the given statement is a set

A set is a collection of well-defined objects.

We can identify integers that are all the collection of even integers. So it is not a well-defined object.

Thus, the collection of all even integers is a set.

viii. The collection of questions in this chapter.

Ans-

To determine if the given statement is a set

A set is a collection of well-defined objects.

We can easily identify the questions that are in this chapter. So it is a well-defined object.

Thus, the collection of questions in this chapter is a set.

ix. A collection of the most dangerous animals in the world.

Ans-

To determine if the given statement is a set

A set is a collection of well-defined objects.

The criteria for determining the most dangerous animals may vary according to the person. So it is not a well-defined object.

Thus, the collection of the most dangerous animals in the world is a set.

2. Let $A=\left\{ 1,2,3,4,5,6 \right\}$. Insert the appropriate symbol $\in $ or $\notin $ in the blank spaces:

i. $5...A$

Ans-

Given that,

$A=\left\{ 1,2,3,4,5,6, \right\}$

To insert the appropriate symbol $\in $ or $\notin $

The number $5$ is in the set.

$\therefore 5\in A$

ii. $8...A$

Ans-

Given that,

$A=\left\{ 1,2,3,4,5,6, \right\}$

To insert the appropriate symbol $\in $ or $\notin $

The number $8$ is not in the set.

$\therefore 8\notin A$

iii. $0...A$

Ans-

Given that,

$A=\left\{ 1,2,3,4,5,6, \right\}$

To insert the appropriate symbol $\in $ or $\notin $

The number $0$ is not in the set.

$\therefore 0\notin A$

iv. $4...A$

Ans-

Given that,

$A=\left\{ 1,2,3,4,5,6, \right\}$

To insert the appropriate symbol $\in $ or $\notin $

The number $4$ is in the set.

$\therefore 4\in A$

v. $2...A$

Ans-

Given that,

$A=\left\{ 1,2,3,4,5,6, \right\}$

To insert the appropriate symbol $\in $ or $\notin $

The number $2$ is in the set.

$\therefore 2\in A$

vi. $10...A$

Ans-

Given that,

$A=\left\{ 1,2,3,4,5,6, \right\}$

To insert the appropriate symbol $\in $ or $\notin $

The number $10$ is not in the set.

$\therefore 10\notin A$

3. Write the following sets in roster form:

i. $A=\left\{ x:x\text{ is an integer and -3x7} \right\}$

Ans-

Given that,

$A=\left\{ x:x\text{ is an integer and -3x7} \right\}$

To write the above expression in its roaster form

In roaster form, the order in which the elements are listed is immaterial.

The elements of the set are $-2,-1,0,1,2,3,4,5,6$.

$\therefore $The roaster form of the set $A=\left\{ x:x\text{ is an integer and -3x7} \right\}$ is $A=\left\{ -2,-1,0,1,2,3,4,5,6 \right\}$.

ii. $B=\left\{ x:x\text{ is a natural number less than 6} \right\}$

Ans-

Given that,

$B=\left\{ x:x\text{ is a natural number less than 6} \right\}$

To write the above expression in its roaster form

In roaster form, the order in which the elements are listed is immaterial.

The elements of the set are $1,2,3,4,5$.

$\therefore $The roaster form of the set $B=\left\{ x:x\text{ is a natural number less than 6} \right\}$ is $B=\left\{ 1,2,3,4,5 \right\}$.

iii. $C=\left\{ x:x\text{ is a two-digit natural number such that sum of its digits is 8} \right\}$

Ans-

Given that,

$C=\left\{ x:x\text{ is a two-digit natural number such that sum of its digits is 8} \right\}$

To write the above expression in its roaster form

In roaster form, the order in which the elements are listed is immaterial.

The elements of the set are $17,26,35,44,53,62,71,80$ such that their sum is $8$

$\therefore $The roaster form of the set $C=\left\{ x:x\text{ is a two-digit natural number such that sum of its digits is 8} \right\}$ is $\left\{ 17,26,35,44,53,62,71,80 \right\}$.

iv. $D=\left\{ x:x\text{ is a prime number which is divisor of 60} \right\}$

Ans-

Given that,

$D=\left\{ x:x\text{ is a prime number which is divisor of 60} \right\}$

To write the above expression in its roaster form

In roaster form, the order in which the elements are listed is immaterial.

The divisors of $60$ are $2,3,4,5,6$. Among these, the prime numbers are $2,3,5$

The elements of the set are $2,3,5$.

$\therefore $The roaster form of the set $D=\left\{ x:x\text{ is a prime number which is divisor of 60} \right\}$ is $D=\left\{ 2,3,5 \right\}$.

v. $E=$ The set of all letters in the word TRIGONOMETRY

Ans-

Given that,

$E=$The set of all letters in the word TRIGONOMETRY

To write the above expression in its roaster form

In roaster form, the order in which the elements are listed is immaterial.

There are $12$ letters in the word TRIGONOMETRY out of which T, R and O gets repeated.

The elements of the set are T, R, I G, O, N, M, E, Y.

$\therefore $The roaster form of the set $E=$The set of all letters in the word TRIGONOMETRY is $E=\left\{ T,R,I,G,O,N,M,E,Y \right\}$.

vi. $F=$The set of all letters in the word BETTER

Ans-

Given that,

$F=$The set of all letters in the word BTTER

To write the above expression in its roaster form

In roaster form, the order in which the elements are listed is immaterial.

There are $6$ letters in the word BETTER out of which E and T are repeated.

The elements of the set are B, E, T, R.

$\therefore $The roaster form of the set $F=$The set of all letters in the word BTTER

 is $F=\left\{ B,E,T,R \right\}$.

4. Write the following sets in the set builder form:

i. $\left( 3,6,9,12 \right)$

Ans-

Given that,

$\left\{ 3,6,9,12 \right\}$

To represent the given set in the set builder form

In set-builder form, all the elements of a set possess a single common property that is not possessed by any element outside the set.

From the given set, we observe that the numbers in the set are multiple of $3$ from $1$ to $4$ such that $\left\{ x:x=3n,n\in N\text{ and 1}\le \text{n}\le \text{4} \right\}$

$\therefore \left\{ 3,6,9,12 \right\}=\left\{ x:x=3n,n\in N\text{ and 1}\le \text{n}\le \text{4} \right\}$

ii. $\left\{ 2,4,8,16,32 \right\}$

Ans-

Given that,

$\left\{ 2,4,8,16,32 \right\}$

To represent the given set in the set builder form

In set-builder form, all the elements of a set possess a single common property which is not possessed by any element outside the set.

From the given set, we observe that the numbers in the set are powers of $2$from $1$ to $5$ such that $\left\{ x:x={{2}^{n}},n\in N\text{ and 1}\le \text{n}\le 5 \right\}$

$\therefore \left\{ 2,4,8,16,32 \right\}=\left\{ x:x={{2}^{n}},n\in N\text{ and 1}\le \text{n}\le 5 \right\}$

iii. $\left\{ 5,25,125,625 \right\}$

Ans-

Given that,

$\left\{ 5,25,125,625 \right\}$

To represent the given set in the set builder form

In set builder form, all the elements of a set possess a single common property which is not possessed by any element outside the set.

From the given set, we observe that the numbers in the set are powers of $5$from $1$ to $4$ such that $\left\{ x:x={{5}^{n}},n\in N\text{ and 1}\le \text{n}\le \text{4} \right\}$

$\therefore \left\{ 5,25,125,625 \right\}=\left\{ x:x={{5}^{n}},n\in N\text{ and 1}\le \text{n}\le \text{4} \right\}$

iv. $\left\{ 2,4,6,... \right\}$

Ans-

Given that,

$\left\{ 2,4,6,... \right\}$

To represent the given set in the set builder form

In set builder form, all the elements of a set possess a single common property which is not possessed by any element outside the set.

From the given set, we observe that the numbers are the set of all even natural numbers.

$\therefore \left\{ 2,4,6,... \right\}=\left\{ x:x\text{ is an even natural number} \right\}$

v) $\left\{ 1,4,9,...100 \right\}$

Ans-

Given that,

$\left\{ 1,4,9,...100 \right\}$

To represent the given set in the set builder form

In set builder form, all the elements of a set possess a single common property which is not possessed by any element outside the set.

From the given set, we observe that the numbers in the set squares of numbers form $1$ to $10$ such that $\left\{ x:x={{n}^{2}},n\in N\text{ and 1}\le \text{n}\le 10 \right\}$

$\therefore \left\{ 1,4,9,...100 \right\}=\left\{ x:x={{n}^{2}},n\in N\text{ and 1}\le \text{n}\le 10 \right\}$

5. List all the elements of the following sets:

i. $A=\left\{ x:x\text{ is an odd natural number} \right\}$

Ans-

Given that,

$A=\left\{ x:x\text{ is an odd natural number} \right\}$

To list the elements of the given set

The odd natural numbers are $1,3,5,...$

$\therefore $The set $A=\left\{ x:x\text{ is an odd natural number} \right\}$ has the odd natural numbers that are $\left\{ 1,3,5,... \right\}$

ii. $B=\left\{ x:x\text{ is an integer;-}\frac{1}{2}<x<\frac{9}{2} \right\}$

Ans-

Given that,

$B=\left\{ x:x\text{ is an integer;-}\frac{1}{2}<x<\frac{1}{2} \right\}$

To list the elements of the given set

$-\frac{1}{2}=-0.5$ and $\frac{9}{2}=4.5$

So the integers between $-0.5$ and $4.5$ are $0,1,2,3,4$

$\therefore $The set $B=\left\{ x:x\text{ is an integer;-}\frac{1}{2}<x<\frac{1}{2} \right\}$ has an integers that are between $\left\{ 0,1,2,3,4 \right\}$


iii. $C=\left\{ x:x\text{ is an integer;}{{\text{x}}^{2}}\le 4 \right\}$

Ans-

Given that,

$C=\left\{ x:x\text{ is an integer;}{{\text{x}}^{2}}\le 4 \right\}$

To list the elements of the given set

It is observed that,

${{x}^{2}}\le 4$

${{\left( -2 \right)}^{2}}=4\le 4$

${{\left( -1 \right)}^{2}}=1\le 4$

${{\left( 0 \right)}^{2}}=0\le 4$

${{\left( 1 \right)}^{2}}=1\le 4$

${{\left( 2 \right)}^{2}}=4\le 4$

$\therefore $The set $C=\left\{ x:x\text{ is an integer;}{{\text{x}}^{2}}\le 4 \right\}$ contains elements such as $\left\{ -2,-1,0,1,2 \right\}$


iv. $D=\left\{ x:x\text{ is a letter in the word ''LOYAL''} \right\}$

Ans-

Given that,

$D=\left\{ x:x\text{ is a letter in the word ''LOYAL''} \right\}$

To list the elements of the given set

There are $5$ total letters in the given word in which L gets repeated two times.

So the elements in the set are $\left\{ L,O,Y,A \right\}$

$\therefore $The set $D=\left\{ x:x\text{ is a letter in the word ''LOYAL''} \right\}$ consists the elements $\left\{ L,O,Y,A \right\}$.


v. $E=\left\{ x:x\text{ is a month of a year not having 31 days} \right\}$

Ans-

Given that,

$E=\left\{ x:x\text{ is a month of a year not having 31 days} \right\}$

To list the elements of the given set

The months that don’t have $31$ are as follows:

February, April, June, September, November

$\therefore $The set $E=\left\{ x:x\text{ is a month of a year not having 31 days} \right\}$ consist of the elements such that $\left\{ \text{February, April, June, September, November} \right\}$


vi. $F=\left\{ x:x\text{ is a consonant in the English alphabet which precedes k} \right\}$

Ans-

Given that,

$F=\left\{ x:x\text{ is a consonant in the English alphabet which precedes k} \right\}$

To list the elements of the given set

The consonants are letters in English alphabet other than vowels such as a, e, i, o, u and the consonants that precedes k include b, c, d, f, g, h, j

$\therefore $The set $F=\left\{ x:x\text{ is a consonant in the English alphabet which precedes k} \right\}$ consists of the set $\left\{ b,c,d,f,g,h,j \right\}$

6. Match each of the sets on the left in the roaster form with the same set on the right described in set-builder form.

i. $\left\{ 1,2,3,6 \right\}$

Ans-

Given that,

$\left\{ 1,2,3,6 \right\}$

To match the roaster form in the left with the set builder form in the right

In roaster form, the order in which the elements are listed is immaterial.

In set builder form, all the elements of a set possess a single common property which is not possessed by any element outside the set.

It has been observed from the set that these set of numbers are the set of natural numbers which are also the divisors of $6$

Thus, $\left\{ 1,2,3,6 \right\}=\left\{ x:x\text{ is a natural number and is a divisor of 6} \right\}$ is the correct option which is option (c)


ii. $\left\{ 2,3 \right\}$

Ans-

Given that,

$\left\{ 2,3 \right\}$

To match the roaster form in the left with the set builder form in the right

In roaster form, the order in which the elements are listed is immaterial.

In set builder form, all the elements of a set possess a single common property which is not possessed by any element outside the set.

It has been observed from the set that these set of numbers are the set of prime numbers which are also the divisors of $6$

Thus, $\left\{ 2,3 \right\}=\left\{ x:x\text{ is a prime number and is a divisor of 6} \right\}$ is the correct option which is option (a)


iii. $\left\{ M,A,T,H,E,I,C,S \right\}$

Ans-

Given that,

$\left\{ M,A,T,H,E,I,C,S \right\}$

To match the roaster form in the left with the set builder form in the right

In roaster form, the order in which the elements are listed is immaterial.

In set builder form, all the elements of a set possess a single common property which is not possessed by any element outside the set.

It has been observed from the set of these letters of word MATHEMATICS.

Thus, $\left\{ M,A,T,H,E,I,C,S \right\}=\left\{ x:x\text{ is a letter of the word MATHEMATICS} \right\}$ is the correct option which is option (d)


iv. $\left\{ 1,3,5,7,9 \right\}$

Ans-

Given that,

$\left\{ 1,3,5,7,9 \right\}$

To match the roaster form in the left with the set builder form in the right

In roaster form, the order in which the elements are listed is immaterial.

In set builder form, all the elements of a set possess a single common property which is not possessed by any element outside the set.

It has been observed from the set that these sets of numbers are the set of odd numbers that are less than $10$.

Thus, $\left\{ 1,3,5,7,9 \right\}=\left\{ x:x\text{ is a odd number less than 10} \right\}$ is the correct option which is option (b)

Exercise (1.2)

1. Which of the following are examples of the null set

i. Set of odd natural numbers divisible by $2$

Ans-

Given that,

Set of odd natural numbers divisible by $2$

To find if the given statement is an example of null set

A set which does not contain any element is called the empty set or the null set or the void set.

There is no odd number that will be divisible by $2$ and so this set is a null set.

$\therefore $The set of odd natural numbers divisible by $2$ is a null set.

ii. Set of even prime numbers

Ans-

Given that,

Set of even prime numbers.

To find if the given statement is an example of null set

A set which does not contain any element is called the empty set or the null set or the void set.

There was an even number $2$, which will be the one and only even prime number. So the set contains an element. So it is not a null set.

$\therefore $The set of even prime numbers is not a null set.

iii. $\left\{ x:x\text{ is a natural numbers, x5 and x7} \right\}$

Ans-

Given that,

$\left\{ x:x\text{ is a natural numbers, x5 and x7} \right\}$

To find if the given statement is an example of null set

A set which does not contain any element is called the empty set or the null set or the void set.

There was no  number that will be less than $5$ and greater than $7$ simultaneously. So it is a null set

$\therefore $  $\left\{ x:x\text{ is a natural numbers, x5 and x7} \right\}$ is a null set

iv. $\left\{ y:y\text{ is a point common to any two parallel lines} \right\}$

Ans-

Given that,

$\left\{ y:y\text{ is a point common to any two parallel lines} \right\}$

To find if the given statement is an example of null set

A set which does not contain any element is called the empty set or the null set or the void set.

The parallel lines do not intersect each other. So it does not have a common point of intersection. So it is a null set.

$\therefore $  $\left\{ y:y\text{ is a point common to any two parallel lines} \right\}$is a null set.

2. Which of the following sets are finite or infinite.

i. The sets of months of a year

Ans-

Given that,

The sets of months of a year

To find if the set is finite or infinite

A set which is empty or consists of a definite number of elements is called finite otherwise the set is called infinite.

A year has twelve months which has defined number of elements

$\therefore $The set of months of a year is finite.

ii. $\left\{ 1,2,3... \right\}$

Ans-

Given that,

$\left\{ 1,2,3... \right\}$

To find if the set is finite or infinite

A set which is empty or consists of a definite number of elements is called finite otherwise the set is called infinite.

The set consists of an infinite number of natural numbers.

$\therefore $The set $\left\{ 1,2,3... \right\}$ is infinite since it contains an infinite number of elements.

iii. $\left\{ 1,2,3,...,99,100 \right\}$

Ans-

Given that,

$\left\{ 1,2,3,...,99,100 \right\}$

To find if the set is finite or infinite

A set which is empty or consists of a definite number of elements is called finite otherwise the set is called infinite.

This set contains the elements from $1$ to $100$which are finite in number.

$\therefore $The set $\left\{ 1,2,3,...,99,100 \right\}$ is finite.

iv. The set of positive integers greater than $100$

Ans-

Given that,

The set of positive integers greater than $100$

To find if the set is finite or infinite

A set which is empty or consists of a definite number of elements is called finite otherwise the set is called infinite.

The positive integers which are greater than $100$ are infinite in number.

$\therefore $The set of positive integers greater than $100$ is infinite.

v. The set of prime numbers less than $99$

Ans-

Given that,

The set of prime numbers less than $99$

To find if the set is finite or infinite

A set which is empty or consists of a definite number of elements is called finite otherwise the set is called infinite.

The prime numbers less than $99$ are finite in number.

$\therefore $The set of prime numbers less than $99$ is finite.

3. State whether each of the following set is finite or infinite:

i. The sets of lines which are parallel to $x$ axis

Ans-

Given that,

The set of lines which are parallel to $x$ axis

To find if the set is finite or infinite

A set which is empty or consists of a definite number of elements is called finite otherwise the set is called infinite.

The lines parallel to the $x$ axis are infinite in number.

$\therefore $The set of lines parallel to $x$ axis is infinite.

ii. The set of letters in English alphabet

Ans-

Given that,

The set of letter sin English alphabet

To find if the set is finite or infinite

A set which is empty or consists of a definite number of elements is called finite otherwise the set is called infinite.

English alphabet consist of $26$ elements which is finite in number

$\therefore $The set of letters in the English alphabet is finite.

iii. The set of numbers which are multiple of $5$

Ans-

Given that,

The set of numbers which are multiple of $5$

To find if the set is finite or infinite

A set which is empty or consists of a definite number of elements is called finite otherwise the set is called infinite.

The numbers which are all multiple of $5$ are infinite in number.

$\therefore $The set of numbers which are multiple of $5$is infinite.

iv. The set of animals living on the earth

Ans-

Given that,

The set of animals living on the earth

To find if the set is finite or infinite

A set which is empty or consists of a definite number of elements is called finite otherwise the set is called infinite.

Although the number of animals on the earth is quite a big number, it is finite.

$\therefore $The set of animals living on the earth is finite.

v. The set of circles passing through the origin $\left( 0,0 \right)$

Ans-

Given that,

The set of circles passing through the origin $\left( 0,0 \right)$

To find if the set is finite or infinite

A set which is empty or consists of a definite number of elements is called finite otherwise the set is called infinite.

The number of circles passing through the origin may be infinite in number.

$\therefore $The set of circles passing through origin $\left( 0,0 \right)$ is infinite.

4. In the following, state whether $A=B$ or not

i. $A=\left\{ a,b,c,d \right\};B=\left\{ d,c,b,a \right\}$

Ans-

Given that,

$A=\left\{ a,b,c,d \right\};B=\left\{ d,c,b,a \right\}$

To state whether $A=B$

We know that the order in which the elements are listed are insignificant. So $A=B$

$\therefore A=B$

ii. $A=\left\{ 4,8,12,16 \right\}:B=\left\{ 8,4,16,18 \right\}$

Ans-

Given that,

$A=\left\{ 4,8,12,16 \right\}:B=\left\{ 8,4,16,18 \right\}$

To state whether $A=B$

We know that $12\in A$ but $12\notin B$

$\therefore A\ne B$

iii. $A=\left\{ 2,4,6,8,10 \right\};B=\left\{ x:x\text{ is a positive integer and x}\le \text{10} \right\}$

Ans-

Given that,

$A=\left\{ 2,4,6,8,10 \right\};B=\left\{ x:x\text{ is a positive integer and x}\le \text{10} \right\}$

To state whether $A=B$

$A=\left\{ 2,4,6,8,10 \right\}$

The positive integers less than $10$ are $B=\left\{ 2,4,6,8,10 \right\}$ So $A=B$

$\therefore A=B$

iv. $A=\left\{ x:x\text{ is a multiple of 10} \right\};B=\left\{ 10,15,20,25,30,... \right\}$

Ans-

Given that,

$A=\left\{ x:x\text{ is a multiple of 10} \right\};B=\left\{ 10,15,20,25,30,... \right\}$

To state whether $A=B$

$A=\left\{ 10,20,30,40,... \right\}$

$B=\left\{ 10,15,20,25,30,... \right\}$

The elements of A consists only of multiples of $10$ and not of $5$. So $A\ne B$

$\therefore A\ne B$

5. Are the following pair of sets equal? Give reasons.

i. $A=\left\{ 2,3 \right\};B=\left\{ x:x\text{ is solution of }{{\text{x}}^{2}}+5x+6=0 \right\}$

Ans-

Given that,

$A=\left\{ 2,3 \right\};B=\left\{ x:x\text{ is a solution of }{{\text{x}}^{2}}+5x+6=0 \right\}$

To state whether $A=B$

Solving ${{x}^{2}}+5x+6=0$,

${{x}^{2}}+3x+2x+6=0$

$\left( x+2 \right)\left( x+3 \right)=0$

$x=-2,-3$

$B=\left\{ -2,-3 \right\}$ and $A=\left\{ 2,3 \right\}$

So $A\ne B$

$\therefore A\ne B$

ii. $A=\left\{ x:x\text{ is a letter in the word FOLLOW} \right\};B=\left\{ y:y\text{ is a letter in the word WOLF} \right\}$

Ans-

Given that,

$A=\left\{ x:x\text{ is a letter in the word FOLLOW} \right\};B=\left\{ y:y\text{ is a letter in the word WOLF} \right\}$

To state whether $A=B$

$A=\left\{ x:x\text{ is a letter in the word FOLLOW} \right\}=\left\{ F,O,L,W \right\}$

$B=\left\{ y:y\text{ is a letter in the word WOLF} \right\}=\left\{ W,O,L,F \right\}$

We know that the order in which the elements are listed are insignificant. So $A=B$

$\therefore A=B$

6. From the sets given below, select equal sets:

$A=\left\{ 2,4,8,12 \right\},B=\left\{ 1,2,3,4 \right\},C=\left\{ 4,8,12,14 \right\},D=\left\{ 3,1,4,2 \right\}$  $E=\left\{ -1,1 \right\},F=\left\{ 0,a \right\},G=\left\{ 1,-1 \right\},H=\left\{ 0,1 \right\}$

Ans-

Given that,

$A=\left\{ 2,4,8,12 \right\},B=\left\{ 1,2,3,4 \right\},C=\left\{ 4,8,12,14 \right\},D=\left\{ 3,1,4,2 \right\}$

$E=\left\{ -1,1 \right\},F=\left\{ 0,a \right\},G=\left\{ 1,-1 \right\},H=\left\{ 0,1 \right\}$

To select equal sets from the given set

Two sets A and B are said to be equal if they have exactly the same elements and we write A = B

We can observe from the sets that,

$8\in A,8\notin B,8\notin D,8\notin E,8\notin F,8\notin G,8\notin H$

And thus 

$A\ne B,A\ne D,A\ne E,A\ne F,A\ne G,A\ne H$

But $8\in C$

And checking other elements,

$2\in A,2\notin C$

So $A\ne C$

$3\in B,3\notin C,3\notin E,3\notin F,3\notin G,3\notin H$

And thus,

$B\ne C,B\ne E,B\ne F,B\ne G,B\ne H$

$12\in C,12\notin D,12\notin E,12\notin F,12\notin G,12\notin H$

And thus 

$C\ne D,C\ne E,C\ne F,C\ne G,C\ne H$

$4\in D,4\notin E,4\notin F,4\notin G,4\notin H$

And thus,

$D\ne E,D\ne F,D\ne G,D\ne H$

Similarly $E\ne F,E\ne G,E\ne H$

$F\ne G,F\ne H$

$G\ne H$

We know that the order of the elements I listed are insignificant.

So $B=D,E=G$

$\therefore $He equal sets are $B=D$ and $E=G$

Exercise (1.3)

1. Make correct statements by filling in the symbols $\subset $ or $\not\subset $ in the blank spaces.

i. $\left\{ 2,3,4 \right\}...\left\{ 1,2,3,4,5 \right\}$

Ans-

Given that,

$\left\{ 2,3,4 \right\}...\left\{ 1,2,3,4,5 \right\}$
To fill in the correct symbols $\subset $ or $\not\subset $ inn the blank spaces

A set A is said to be a subset of B if every element of A is also an element of B

$A\subset B$ if $a\in A,a\in B$

The element in the set $\left\{ 2,3,4 \right\}$ is also in the set $\left\{ 1,2,3,4,5 \right\}$

$\therefore \left\{ 2,3,4 \right\}\subset \left\{ 1,2,3,4,5 \right\}$


ii. $\left\{ a,b,c \right\}...\left\{ b,c,d \right\}$

Ans-

Given that,

$\left\{ a,b,c \right\}...\left\{ b,c,d \right\}$
To fill in the correct symbols $\subset $ or $\not\subset $ inn the blank spaces

A set A is said to be a subset of B if every element of A is also an element of B

$A\subset B$ if $a\in A,a\in B$

The element in the set $\left\{ a,b,c \right\}$ is not in the set $\left\{ b,c,d \right\}$

$\therefore \left\{ a,b,c \right\}\not\subset \left\{ b,c,d \right\}$


iii. $\left\{ x:x\text{ is a student of class XI of your school} \right\}...$

$\left\{ x:x\text{ is a student of your school} \right\}$

Ans-

Given that,

$\left\{ x:x\text{ is a student of class XI of your school} \right\}...$

$\left\{ x:x\text{ is a student of your school} \right\}$
To fill in the correct symbols $\subset $ or $\not\subset $ inn the blank spaces

A set A is said to be a subset of B if every element of A is also an element of B

$A\subset B$ if $a\in A,a\in B$

The set of students of class XI would also be inside the set of students in school

$\backepsilon  \left\{ x:x\text{ is a student of class XI of your school} \right\}\subset $  $\left\{ x:x\text{ is a student of your school} \right\}$

iv. $\left\{ x:x\text{ is a circle in the plane } \right\}...$

$\left\{ x:x\text{ is a circle in the same plane with radius 1 unit} \right\}$

Ans-

Given that,

$\left\{ x:x\text{ is a circle in the plane } \right\}...$

$\left\{ x:x\text{ is a circle in the same plane with radius 1 unit} \right\}$
To fill in the correct symbols $\subset $ or $\not\subset $ inn the blank spaces

A set A is said to be a subset of B if every element of A is also an element of B

$A\subset B$ if $a\in A,a\in B$

The set of circles in the plane with a unit radius will be in the set of the circles in the same plane. So the set of circles in the plane is not in the set of circles with unit radius in the same plane.

$\therefore \left\{ x:x\text{ is a circle in the plane } \right\}\not\subset $  $\left\{ x:x\text{ is a circle in the same plane with radius 1 unit} \right\}$

v. $\left\{ x:x\text{ is a triangle in the plane} \right\}...$

$\left\{ x:x\text{ is a rectangle in the plane} \right\}$

Ans-

Given that,

$\left\{ x:x\text{ is a triangle in the plane} \right\}...$

$\left\{ x:x\text{ is a rectangle in the plane} \right\}$

To fill in the correct symbols $\subset $ or $\not\subset $ inn the blank spaces

A set A is said to be a subset of B if every element of A is also an element of B

$A\subset B$ if $a\in A,a\in B$

From the given expression itself, we know that the set of triangles in the plane are not in the set of rectangles in the plane.

$\therefore \left\{ x:x\text{ is a triangle in the plane} \right\}\not\subset $  $\left\{ x:x\text{ is a rectangle in the plane} \right\}$

vi. $\left\{ x:x\text{ is an equilateral triangle in the plane} \right\}...$

$\left\{ x:x\text{ is a triangle in the plane} \right\}$

Ans-

Given that,

$\left\{ x:x\text{ is an equilateral triangle in the plane} \right\}...$  $\left\{ x:x\text{ is a triangle in the plane} \right\}$

To fill in the correct symbols $\subset $ or $\not\subset $ inn the blank spaces

A set A is said to be a subset of B if every element of A is also an element of B

$A\subset B$ if $a\in A,a\in B$

From the above expression, we know that the set of equilateral triangles in the plane is in the set of triangles in the same plane

$\therefore \left\{ x:x\text{ is an equilateral triangle in the plane} \right\}\subset $  $\left\{ x:x\text{ is a triangle in the plane} \right\}$


vii. $\left\{ x:x\text{ is an even natural number} \right\}...\left\{ x:x\text{ is an integer} \right\}$

Ans-

Given that,

$\left\{ x:x\text{ is an even natural number} \right\}...\left\{ x:x\text{ is an integer} \right\}$
To fill in the correct symbols $\subset $ or $\not\subset $ inn the blank spaces

A set A is said to be a subset of B if every element of A is also an element of B

$A\subset B$ if $a\in A,a\in B$

The set of even natural numbers are in the set of integers.

$\therefore \left\{ x:x\text{ is an even natural number} \right\}\subset \left\{ x:x\text{ is an integer} \right\}$

2. Examine whether the following statements are true or false

i. $\left\{ a,b \right\}\not\subset \left\{ b,c,a \right\}$

Ans-

Given that,

$\left\{ a,b \right\}\not\subset \left\{ b,c,a \right\}$
To examine whether the above statement is true or false

A set A is said to be a subset of B if every element of A is also an element of B

$A\subset B$ if $a\in A,a\in B$

The element in the set $\left\{ a,b \right\}$ is also in the set $\left\{ b,c,a \right\}$

$\therefore \left\{ a,b \right\}\subset \left\{ b,c,a \right\}$

$\therefore $The given statement is false

ii. $\left\{ a,e \right\}\subset \left\{ x:x\text{ is an vowel in English alpahbet} \right\}$

Ans-

Given that,

$\left\{ a,e \right\}\subset \left\{ x:x\text{ is an vowel in English alpahbet} \right\}$

To examine whether the above statement is true or false

A set A is said to be a subset of B if every element of A is also an element of B

$A\subset B$ if $a\in A,a\in B$

The element in the set $\left\{ a,e \right\}$ is also in the set $\left\{ a,e,i,o,u \right\}$

$\therefore \left\{ a,e \right\}\subset \left\{ x:x\text{ is an vowel in English alpahbet} \right\}$

$\therefore $The given statement is true.

ii. $\left\{ 1,2,3 \right\}\subset \left\{ 1,3,5 \right\}$

Ans-

Given that,

$\left\{ 1,2,3 \right\}\subset \left\{ 1,3,5 \right\}$
To examine whether the above statement is true or false

A set A is said to be a subset of B if every element of A is also an element of B

$A\subset B$ if $a\in A,a\in B$

The element in the set $\left\{ 1,2,3 \right\}$ is not in the set $\left\{ 1,3,5 \right\}$ since $2\in \left\{ 1,2,3 \right\}$ and $2\notin \left\{ 1,3,5 \right\}$

$\left\{ 1,2,3 \right\}\not\subset \left\{ 1,3,5 \right\}$

$\therefore $The given statement is false.


iii. $\left\{ a \right\}\subset \left\{ a,b,c \right\}$

Ans-

Given that,

$\left\{ a \right\}\subset \left\{ a,b,c \right\}$

To examine whether the above statement is true or false

A set A is said to be a subset of B if every element of A is also an element of B

$A\subset B$ if $a\in A,a\in B$

The element in the set $\left\{ a \right\}$ is also in the set $\left\{ a,b,c \right\}$

$\therefore \left\{ a \right\}\subset \left\{ a,b,c \right\}$

$\therefore $The given statement is true.

iv. $\left\{ a \right\}\in \left\{ a,b,c \right\}$

Ans-

Given that,

$\left\{ a \right\}\in \left\{ a,b,c \right\}$
To examine whether the above statement is true or false

A set A is said to be a subset of B if every element of A is also an element of B

$A\subset B$ if $a\in A,a\in B$

The element in the set $\left\{ a \right\}$ and the elements in the set $\left\{ a,b,c \right\}$ are $a,b,c$

$\therefore \left\{ a \right\}\subset \left\{ a,b,c \right\}$

$\therefore $The given statement is false.


v. $\left\{ x:x\text{ is an even natural less than 6} \right\}\subset $  $\left\{ x:x\text{ is a natural number which divide 36} \right\}$

Ans-

Given that,

$\left\{ x:x\text{ is an even natural less than 6} \right\}\subset $  $\left\{ x:x\text{ is a natural number which divide 36} \right\}$To examine whether the above statement is true or false

A set A is said to be a subset of B if every element of A is also an element of B

$A\subset B$ if $a\in A,a\in B$

$\left\{ x:x\text{ is an even natural less than 6} \right\}=\left\{ 2,4 \right\}$

$\left\{ x:x\text{ is a natural number which divide 36} \right\}=\left\{ 1,2,3,4,6,9,12,18,36 \right\}$  $\therefore \left\{ x:x\text{ is an even natural less than 6} \right\}\subset $  $\left\{ x:x\text{ is a natural number which divide 36} \right\}$

$\therefore $The given statement is true.


3. Let $A=\left\{ 1,2,\left\{ 3,4 \right\},5 \right\}$. Which of the following statements are incorrect and why?

i. $\left\{ 3,4 \right\}\subset A$

Ans-

Given that,

$A=\left\{ 1,2,\left\{ 3,4 \right\},5 \right\}$

To find if $\left\{ 3,4 \right\}\subset A$ is correct or incorrect.

A set A is said to be a subset of B if every element of A is also an element of B

$A\subset B$ if $a\in A,a\in B$

From the above statement,

$3\in \left\{ 3,4 \right\}$, however $3\notin A$

$\therefore $The given statement $\left\{ 3,4 \right\}\subset A$ is incorrect

ii. $\left\{ 3,4 \right\}\in A$

Ans-

Given that,

$A=\left\{ 1,2,\left\{ 3,4 \right\},5 \right\}$

To find if $\left\{ 3,4 \right\}\in A$ is correct or incorrect.

From the above statement,

$\left\{ 3,4 \right\}$ is an element of A.

$\therefore \left\{ 3,4 \right\}\in A$

$\therefore $The given statement is correct.

iii. $\left\{ \left\{ 3,4 \right\} \right\}\subset A$

Ans-

Given that,

$A=\left\{ 1,2,\left\{ 3,4 \right\},5 \right\}$

To find if $\left\{ \left\{ 3,4 \right\} \right\}\subset A$ is correct or incorrect.

A set A is said to be a subset of B if every element of A is also an element of B

$A\subset B$ if $a\in A,a\in B$

From the above statement,

$\left\{ 3,4 \right\}\in \left\{ \left\{ 3,4 \right\} \right\}$ so that $\left\{ \left\{ 3,4 \right\} \right\}\in A$

$\therefore \left\{ \left\{ 3,4 \right\} \right\}\subset A$

$\therefore $The given statement $\left\{ \left\{ 3,4 \right\} \right\}\subset A$ is correct.


iv. $1\in A$

Ans-

Given that,

$A=\left\{ 1,2,\left\{ 3,4 \right\},5 \right\}$

To find if $1\in A$ is correct or incorrect.

From the above statement,

$1$ is an element of A.

$\therefore $The statement $1\in A$ is a correct statement.


v. $1\subset A$

Ans-

Given that,

$A=\left\{ 1,2,\left\{ 3,4 \right\},5 \right\}$

To find if $1\subset A$ is correct or incorrect.

A set A is said to be a subset of B if every element of A is also an element of B

$A\subset B$ if $a\in A,a\in B$

From the above statement,

An element of a set can never be a subset of itself. So $1\not\subset A$

$\therefore $The given statement $1\subset A$ is an incorrect statement.


vi. $\left\{ 1,2,5 \right\}\subset A$

Ans-

Given that,

$A=\left\{ 1,2,\left\{ 3,4 \right\},5 \right\}$

To find if $\left\{ 1,2,5 \right\}\subset A$ is correct or incorrect.

A set A is said to be a subset of B if every element of A is also an element of B

$A\subset B$ if $a\in A,a\in B$

From the above statement,

The each element of $\left\{ 1,2,5 \right\}$ is also an element of A, So $\left\{ 1,2,5 \right\}\subset A$

$\therefore $The given statement $\left\{ 1,2,5 \right\}\subset A$ is a correct statement 


vii. $\left\{ 1,2,5 \right\}\in A$

Ans-

Given that,

$A=\left\{ 1,2,\left\{ 3,4 \right\},5 \right\}$

To find if $\left\{ 1,2,5 \right\}\subset A$ is correct or incorrect.

From the above statement,

Element of $\left\{ 1,2,5 \right\}$ is not an element of A, So $\left\{ 1,2,5 \right\}\notin A$

So the given statement $\left\{ 1,2,5 \right\}\in A$ is an incorrect statement.


viii. $\left\{ 1,2,3 \right\}\subset A$

Ans-

Given that,

$A=\left\{ 1,2,\left\{ 3,4 \right\},5 \right\}$

To find if $\left\{ 1,2,3 \right\}\subset A$ is correct or incorrect.

A set A is said to be a subset of B if every element of A is also an element of B

$A\subset B$ if $a\in A,a\in B$

From the above statement, we notice that,

$3\in \left\{ 1,2,3 \right\}$but $3\notin A$

$\left\{ 1,2,3 \right\}\not\subset A$

$\therefore $The given statement $\left\{ 1,2,3 \right\}\subset A$ is an incorrect statement.

ix. $\varnothing \in A$

Ans-

Given that,

$A=\left\{ 1,2,\left\{ 3,4 \right\},5 \right\}$

To find if $\varnothing \in A$ is correct or incorrect.

A set A is said to be a subset of B if every element of A is also an element of B

$A\subset B$ if $a\in A,a\in B$

From the above statement,

$\varnothing $ is not an element of A. So, $\varnothing \notin A$

$\therefore $The given statement $\varnothing \in A$ is an incorrect statement.

x. $\varnothing \subset A$

Ans-

Given that,

$A=\left\{ 1,2,\left\{ 3,4 \right\},5 \right\}$

To find if $\varnothing \subset A$ is correct or incorrect.

A set A is said to be a subset of B if every element of A is also an element of B

$A\subset B$ if $a\in A,a\in B$

From the above statement,

Since $\varnothing $  is a subset of every set, $\varnothing \subset A$

$\therefore $The given statement $\varnothing \subset A$ is a correct statement.


xi. $\left\{ \varnothing  \right\}\subset A$

Ans-

Given that,

$A=\left\{ 1,2,\left\{ 3,4 \right\},5 \right\}$

To find if $\left\{ \varnothing  \right\}\subset A$ is correct or incorrect.

A set A is said to be a subset of B if every element of A is also an element of B

$A\subset B$ if $a\in A,a\in B$

From the above statement,

$\varnothing $ is an element of A and it is not a subset of A. 

$\therefore $The given statement $\left\{ \varnothing  \right\}\subset A$ is an incorrect statement.

3. Write down all the subsets of the following sets:

i. $\left\{ a \right\}$

Ans-

Given that,

$\left\{ a \right\}$

To write the subset of the given sets

A set A is said to be a subset of B if every element of A is also an element of B

$A\subset B$ if $a\in A,a\in B$

Subsets of $\left\{ a \right\}$ are $\varnothing $ and $\left\{ a \right\}$


ii. $\left\{ a,b \right\}$

Ans-

Given that,

$\left\{ a,b \right\}$

To write the subset of the given sets

A set A is said to be a subset of B if every element of A is also an element of B

$A\subset B$ if $a\in A,a\in B$

Subsets of $\left\{ a,b \right\}$ are $\varnothing $ and $\left\{ a \right\},\left\{ b \right\},\left\{ a,b \right\}$


iii. $\left\{ 1,2,3 \right\}$

Ans-

Given that,

$\left\{ 1,2,3 \right\}$

To write the subset of the given sets

A set A is said to be a subset of B if every element of A is also an element of B

$A\subset B$ if $a\in A,a\in B$

Subsets of $\left\{ 1,2,3 \right\}$ are $\varnothing $,$\left\{ 1 \right\},\left\{ 2 \right\},\left\{ 3 \right\},\left\{ 1,2 \right\},\left\{ 2,3 \right\},\left\{ 1,3 \right\},\left\{ 1,2,3 \right\}$


iv. $\varnothing $

Ans-

Given that,

$\varnothing $

To write the subset of the given sets

A set A is said to be a subset of B if every element of A is also an element of B

$A\subset B$ if $a\in A,a\in B$

Subsets of $\varnothing $ is $\varnothing $.


4. How many elements has $P\left( A \right)$, if $A=\varnothing $?

Ans-

Given that,

$A=\varnothing $

To find the number of elements does the $P\left( A \right)$ contain

The collection of all subsets of a set A is called a power set of A and is denoted by $P\left( A \right)$.

We know that if $A$ is a set with $m$elements, that is, $n\left( A \right)=m$, then $n\left[ p\left( A \right) \right]={{2}^{m}}$

If $A=\varnothing $ then $n\left( A \right)=0$

$n\left[ P\left( A \right) \right]={{2}^{0}}$

$=1$

$\therefore P\left( A \right)$ has only one element.


5. Write the following as intervals

i. $\left\{ x:x\in R,-4<x\le 6 \right\}$

Ans-

Given that,

$\left\{ x:x\in R,-4<x\le 6 \right\}$

To write the above expression as intervals

The set of real numbers $\left\{ y:a<y<b \right\}$ is called an open interval and is denoted by $\left( a,b \right)$. The interval which contains the end points also is called close interval and is denoted by$\left[ a,b \right]$

$\therefore \left\{ x:x\in R,-4<x\le 6 \right\}=(-4,6]$


ii. $\left\{ x:x\in R,-12<x<-10 \right\}$

Ans-

Given that,

$\left\{ x:x\in R,-12<x<-10 \right\}$

To write the above expression as intervals

The set of real numbers $\left\{ y:a<y<b \right\}$ is called an open interval and is denoted by $\left( a,b \right)$. The interval which contains the end points also is called close interval and is denoted by$\left[ a,b \right]$

$\therefore \left\{ x:x\in R,-12<x<-10 \right\}=\left( -12,-10 \right)$


iii. $\left\{ x:x\in R,0\le x<7 \right\}$

Ans-

Given that,

$\left\{ x:x\in R,0\le x<7 \right\}$

To write the above expression as intervals

The set of real numbers $\left\{ y:a<y<b \right\}$ is called an open interval and is denoted by $\left( a,b \right)$. The interval which contains the end points also is called close interval and is denoted by$\left[ a,b \right]$

$\because \left\{ x:x\in R,0\le x<7 \right\}=[0,7)$


iv. $\left\{ x:x\in R,3\le x\le 4 \right\}$

Ans-

Given that,

$\left\{ x:x\in R,3\le x\le 4 \right\}$

To write the above expression as intervals

The set of real numbers $\left\{ y:a<y<b \right\}$ is called an open interval and is denoted by $\left( a,b \right)$. The interval which contains the end points also is called close interval and is denoted by$\left[ a,b \right]$

$\therefore \left\{ x:x\in R,3\le x\le 4 \right\}=\left[ 3,4 \right]$


6. Write the following intervals in set builder form.

i. $\left( -3,0 \right)$

Ans-

Given that,

$\left( -3,0 \right)$

To write the above interval in set builder form

The set of real numbers $\left\{ y:a<y<b \right\}$ is called an open interval and is denoted by $\left( a,b \right)$. The interval which contains the end points also is called close interval and is denoted by$\left[ a,b \right]$

$\therefore \left( -3,0 \right)=\left\{ x:x\in R,-3<x<0 \right\}$

ii. $\left[ 6,12 \right]$

Ans-

Given that,

$\left[ 6,12 \right]$

To write the above interval in set builder form

The set of real numbers $\left\{ y:a<y<b \right\}$ is called an open interval and is denoted by $\left( a,b \right)$. The interval which contains the end points also is called close interval and is denoted by$\left[ a,b \right]$

$\therefore \left[ 6,12 \right]=\left\{ x:x\in R,6\le x\le 12 \right\}$


iii. $(6,12]$

Ans-

Given that,

$(6,12]$

To write the above interval in set builder form

The set of real numbers $\left\{ y:a<y<b \right\}$ is called an open interval and is denoted by $\left( a,b \right)$. The interval which contains the end points also is called close interval and is denoted by$\left[ a,b \right]$

$\therefore (6,12]=\left\{ x:x\in R,6<x\le 12 \right\}$


iv. $[-23,5)$

Ans-

Given that,

$[-23,5)$

To write the above interval in set builder form

The set of real numbers $\left\{ y:a<y<b \right\}$ is called an open interval and is denoted by $\left( a,b \right)$. The interval which contains the end points also is called close interval and is denoted by$\left[ a,b \right]$

$\therefore [-23,5)=\left\{ x:x\in R,-23\le x<5 \right\}$


7. What universal set(s) would you propose for each of the following:

i. The set of right triangles

Ans-

To propose the universal set for the set of right triangles

For the set of right triangles, the universal set can be the set of all kinds of triangles or the set of polygons.


ii. The set of isosceles triangles

Ans-

To propose the universal set for the set of right triangles

For the set of isosceles triangles, the universal set can be the set of all kinds of triangles or the set of polygons or the set of two dimensional figures.


8. Given the sets $A=\left\{ 1,3,5 \right\},B=\left\{ 2,4,6 \right\}$ and $C=\left\{ 0,2,4,6,8 \right\}$, which of the following may be considered as universal set(s) for all the three sets A, B and C?

i. $\left\{ 0,1,2,3,4,5,6 \right\}$

Ans-

Given that,

$A=\left\{ 1,3,5 \right\},B=\left\{ 2,4,6 \right\},C=\left\{ 0,2,4,6,8 \right\}$

To find if the given set $\left\{ 0,1,2,3,4,5,6 \right\}$ is the universal set of A, B and C

It can be observed that,

$A\subset $  $\left\{ 0,1,2,3,4,5,6 \right\}$

$B\subset $  $\left\{ 0,1,2,3,4,5,6 \right\}$

$C\not\subset $  $\left\{ 0,1,2,3,4,5,6 \right\}$

$\therefore $The set $\left\{ 0,1,2,3,4,5,6 \right\}$ cannot be the universal set for the sets A, B and C


ii. $\varnothing $

Ans-

Given that,

$A=\left\{ 1,3,5 \right\},B=\left\{ 2,4,6 \right\},C=\left\{ 0,2,4,6,8 \right\}$

To find if the given set $\varnothing $ is the universal set of A, B and C

It can be observed that,

$A\not\subset \varnothing $

$B\not\subset \varnothing $

$C\not\subset \varnothing $

$\therefore $The set $\varnothing $ cannot be an universal set for A, B and C.


iii. $\left\{ 0,1,2,3,4,5,6,7,8,9,10 \right\}$

Ans-

Given that,

$A=\left\{ 1,3,5 \right\},B=\left\{ 2,4,6 \right\},C=\left\{ 0,2,4,6,8 \right\}$

To find if the given set $\left\{ 0,1,2,3,4,5,6,7,8,9,10 \right\}$ is the universal set of A, B and C

It can be observe that,

$A\subset $  $\left\{ 0,1,2,3,4,5,6,7,8,9,10 \right\}$

$B\subset $  $\left\{ 0,1,2,3,4,5,6,7,8,9,10 \right\}$

$C\subset $  $\left\{ 0,1,2,3,4,5,6,7,8,9,10 \right\}$

$\therefore $The set $\left\{ 0,1,2,3,4,5,6,7,8,9,10 \right\}$ is the universal set of A, B and C


iv. $\left\{ 1,2,3,4,5,6,7,8 \right\}$

Ans-

Given that,

$A=\left\{ 1,3,5 \right\},B=\left\{ 2,4,6 \right\},C=\left\{ 0,2,4,6,8 \right\}$

To find if the given set $\left\{ 0,1,2,3,4,5,6 \right\}$ is the universal set of A, B and C

It can be observed that,

$A\subset $  $\left\{ 1,2,3,4,5,6,7,8 \right\}$

$B\subset $  $\left\{ 1,2,3,4,5,6,7,8 \right\}$

$C\not\subset $  $\left\{ 1,2,3,4,5,6,7,8 \right\}$

$\therefore $The set $\left\{ 1,2,3,4,5,6,7,8 \right\}$ is not the universal set of A, B and C


Exercise (1.4)

1. Find the union of each of following pair of sets

i. $X=\left\{ 1,3,5 \right\},Y=\left\{ 1,2,3 \right\}$

Ans-

Given that,

$X=\left\{ 1,3,5 \right\},Y=\left\{ 1,2,3 \right\}$

To find the union of two sets

Let A and B be any two sets. The union of A and B is the set which consists of all the elements of A and B.

$X\cup Y=\left\{ 1,3,5 \right\}\cup \left\{ 1,2,3 \right\}$

$\therefore X\cup Y=\left\{ 1,2,3,5 \right\}$

ii. $A=\left\{ a,e,i,o,u \right\},B=\left\{ a,b,c \right\}$

Ans-

Given that,

$A=\left\{ a,e,i,o,u \right\},B=\left\{ a,b,c \right\}$

To find the union of two sets

Let A and B be any two sets. The union of A and B is the set which consists of all the elements of A and B.

$A\cup B=\left\{ a,e,i,o,u \right\}\cup \left\{ a,b,c \right\}$

$\therefore A\cup B=\left\{ a,b,c,e,i,o,u \right\}$

iii. $A=\left\{ x:x\text{ is a natural number an multiple of 3} \right\},$

$B=\left\{ x:x\text{ is a natural number less than 6} \right\}$

Ans-

Given that,

$A=\left\{ x:x\text{ is a natural number an multiple of 3} \right\},$

$B=\left\{ x:x\text{ is a natural number less than 6} \right\}$

To find the union of two sets

Let A and B be any two sets. The union of A and B is the set which consists of all the elements of A and B.

$A=\left\{ x:x\text{ is a natural number an multiple of 3} \right\},$

$=\left\{ 3,6,9,... \right\}$

$B=\left\{ x:x\text{ is a natural number less than 6} \right\}$

$=\left\{ 1,2,3,4,5,6 \right\}$

$A\cup B=\left\{ 3,6,9,... \right\}\cup \left\{ 1,2,3,4,5,6 \right\}$

$=\left\{ 1,2,3,4,5,6,9,12,15... \right\}$

$\therefore A\cup B$  $=\left\{ 1,2,3,4,5,6,9,12,15... \right\}$

iv. $A=\left\{ x:x\text{ is a natural number 1x}\le \text{6} \right\},$

$B=\left\{ x:x\text{ is a natural number 6x10} \right\}$

Ans-

Given that,

$A=\left\{ x:x\text{ is a natural number 1x}\le \text{6} \right\},$  $B=\left\{ x:x\text{ is a natural number 6x10} \right\}$

To find the union of two sets

Let A and B be any two sets. The union of A and B is the set which consists of all the elements of A and B.

$A=\left\{ x:x\text{ is a natural number 1x}\le \text{6} \right\}=\left\{ 2,3,4,5,6 \right\}$

$B=\left\{ x:x\text{ is a natural number 6x10} \right\}=\left\{ 7,8,9 \right\}$

$A\cup B=\left\{ 2,3,4,5,6 \right\}\cup \left\{ 7,8,9 \right\}$

$\therefore A\cup B=\left\{ 2,3,4,5,6,7,8,9 \right\}$


v. $A=\left\{ 1,2,3 \right\},B=\varnothing $

Ans-

Given that,

$A=\left\{ 1,2,3 \right\},B=\varnothing $

To find the union of two sets

Let A and B be any two sets. The union of A and B is the set that consists of all the elements of A and B.

$A\cup B=\left\{ 1,2,3 \right\}\cup \varnothing $

$\therefore A\cup B=\left\{ 1,2,3 \right\}$

2. Let $A=\left\{ a,b \right\}$ and $B=\left\{ a,b,c \right\}$. Is $A\subset B$? What is $A\cup B$?

Ans-

Given that,

$A=\left\{ a,b \right\}$ and $B=\left\{ a,b,c \right\}$ 

To find if $A\subset B$ and $A\cup B$

A set A is said to be a subset of B if every element of A is also an element of B

$A\subset B$ if $a\in A,a\in B$

It can be observed that $A\subset B$

$A\cup B=\left\{ a,b \right\}\cup \left\{ a,b,c \right\}$

$\therefore A\cup B=\left\{ a,b,c \right\}$


3. If A and B are two sets such that $A\subset B$ then what is $A\bigcup B$

Ans-

Given that,

A and B are two sets

To find $A\cup B$ when $A\subset B$

If A and B are two sets such that $A\subset B$, then $A\cup B=B$


4. If $A=\left\{ 1,2,3,4 \right\},B=\left\{ 3,4,5,6 \right\},C=\left\{ 5,6,7,8 \right\}$ and $D=\left\{ 7,8,9,10 \right\}$; find 

i. $A\cup B$

Ans-

Given that,

$A=\left\{ 1,2,3,4 \right\},B=\left\{ 3,4,5,6 \right\},C=\left\{ 5,6,7,8 \right\}$, $D=\left\{ 7,8,9,10 \right\}$

To find,

$A\cup B$

Let A and B be any two sets. The union of A and B is the set which consists of all the elements of A and B.

$A\cup B=\left\{ 1,2,3,4 \right\}\cup \left\{ 3,4,5,6 \right\}$

$\therefore A\cup B=\left\{ 1,2,3,4,5,6 \right\}$


ii. $A\cup C$

Ans-

Given that,

$A=\left\{ 1,2,3,4 \right\},B=\left\{ 3,4,5,6 \right\},C=\left\{ 5,6,7,8 \right\}$, $D=\left\{ 7,8,9,10 \right\}$

To find,

$A\cup C$

Let A and B be any two sets. The union of A and B is the set which consists of all the elements of A and B.

$A\cup C=\left\{ 1,2,3,4 \right\}\cup \left\{ 5,6,7,8 \right\}$

$\therefore A\cup C=\left\{ 1,2,3,4,5,6,7,8 \right\}$


ii. $B\cup C$

Ans-

Given that,

$A=\left\{ 1,2,3,4 \right\},B=\left\{ 3,4,5,6 \right\},C=\left\{ 5,6,7,8 \right\}$, $D=\left\{ 7,8,9,10 \right\}$

To find,

$B\cup C$

Let A and B be any two sets. The union of A and B is the set which consists of all the elements of A and B.

$B\cup C=\left\{ 3,4,5,6 \right\}\cup \left\{ 5,6,7,8 \right\}$

$\therefore B\cup C=\left\{ 3,4,5,6,7,8 \right\}$


iii. $B\cup D$

Ans-

Given that,

$A=\left\{ 1,2,3,4 \right\},B=\left\{ 3,4,5,6 \right\},C=\left\{ 5,6,7,8 \right\}$, $D=\left\{ 7,8,9,10 \right\}$

To find,

$B\cup D$

Let A and B be any two sets. The union of A and B is the set which consists of all the elements of A and B.

$B\cup D=\left\{ 3,4,5,6 \right\}\cup \left\{ 7,8,9,10 \right\}$

$\therefore B\cup D=\left\{ 3,4,5,6,7,8,9,10 \right\}$

iv. $A\cup B\cup C$

Ans-

Given that,

$A=\left\{ 1,2,3,4 \right\},B=\left\{ 3,4,5,6 \right\},C=\left\{ 5,6,7,8 \right\}$, $D=\left\{ 7,8,9,10 \right\}$

To find,

$A\cup B\cup C$

Let A and B be any two sets. The union of A and B is the set which consists of all the elements of A and B.

$A\cup B\cup C=\left\{ 1,2,3,4 \right\}\cup \left\{ 3,4,5,6 \right\}\cup \left\{ 5,6,7,8 \right\}$

$\therefore A\cup B\cup C=\left\{ 1,2,3,4,5,6,7,8 \right\}$


v. $A\cup B\cup D$

Ans-

Given that,

$A=\left\{ 1,2,3,4 \right\},B=\left\{ 3,4,5,6 \right\},C=\left\{ 5,6,7,8 \right\}$, $D=\left\{ 7,8,9,10 \right\}$

To find,

$A\cup B\cup D$

Let A and B be any two sets. The union of A and B is the set which consists of all the elements of A and B.

$A\cup B\cup D=\left\{ 1,2,3,4 \right\}\cup \left\{ 3,4,5,6 \right\}\cup \left\{ 7,8,9,10 \right\}$

$\therefore A\cup B\cup D=\left\{ 1,2,3,4,5,6,7,8,9,10 \right\}$

vi. $B\cup C\cup D$

Ans-

Given that,

$A=\left\{ 1,2,3,4 \right\},B=\left\{ 3,4,5,6 \right\},C=\left\{ 5,6,7,8 \right\}$, $D=\left\{ 7,8,9,10 \right\}$

To find,

$B\cup C\cup D$

Let A and B be any two sets. The union of A and B is the set which consists of all the elements of A and B.

$B\cup C\cup D=\left\{ 3,4,5,6 \right\}\cup \left\{ 5,6,7,8 \right\}\cup \left\{ 7,8,9,10 \right\}$

$\therefore B\cup C\cup D=\left\{ 3,4,5,6,7,8,9,10 \right\}$


5. Find the intersection of each pair of sets:

i. $X=\left\{ 1,3,5 \right\},Y=\left\{ 1,2,3 \right\}$

Ans-

Given that,

$X=\left\{ 1,3,5 \right\},Y=\left\{ 1,2,3 \right\}$

To find the intersection of the given sets

The intersection of sets A and B is the set of all elements which are common to both A and B.

$X\cap Y=\left\{ 1,3,5 \right\}\cup \left\{ 1,2,3 \right\}$

$\therefore X\cap Y=\left\{ 1,3 \right\}$


ii. $A=\left\{ a,e,i,o,u \right\},B=\left\{ a,b,c \right\}$

Ans-

Given that,

$A=\left\{ a,e,i,o,u \right\},B=\left\{ a,b,c \right\}$

To find the intersection of the given sets

The intersection of sets A and B is the set of all elements which are common to both A and B.

$A\cap B=\left\{ a,e,i,o,u \right\}\cup \left\{ a,b,c \right\}$

$\therefore A\cap B=\left\{ a \right\}$


iii. $A=\left\{ x:x\text{ is a natural number an multiple of 3} \right\},$

$B=\left\{ x:x\text{ is a natural number less than 6} \right\}$

Ans-

Given that,

$A=\left\{ x:x\text{ is a natural number an multiple of 3} \right\},$

$B=\left\{ x:x\text{ is a natural number less than 6} \right\}$

To find the intersection of two sets

The intersection of sets A and B is the set of all elements which are common to both A and B.

$A=\left\{ x:x\text{ is a natural number an multiple of 3} \right\},$

$=\left\{ 3,6,9,... \right\}$

$B=\left\{ x:x\text{ is a natural number less than 6} \right\}$

$=\left\{ 1,2,3,4,5,6 \right\}$

$A\cap B=\left\{ 3,6,9,... \right\}\cap \left\{ 1,2,3,4,5,6 \right\}$

$=\left\{ 3 \right\}$

$\therefore A\cap B$  $=\left\{ 3 \right\}$


iv. $A=\left\{ x:x\text{ is a natural number 1x}\le \text{6} \right\},$

$B=\left\{ x:x\text{ is a natural number 6x10} \right\}$

Ans-

Given that,

$A=\left\{ x:x\text{ is a natural number 1x}\le \text{6} \right\},$ 

$B=\left\{ x:x\text{ is a natural number 6x10} \right\}$

To find the intersection of two sets

The intersection of sets A and B is the set of all elements which are common to both A and B.

$A=\left\{ x:x\text{ is a natural number 1x}\le \text{6} \right\}=\left\{ 2,3,4,5,6 \right\}$

$B=\left\{ x:x\text{ is a natural number 6x10} \right\}=\left\{ 7,8,9 \right\}$

$A\cap B=\left\{ 2,3,4,5,6 \right\}\cap \left\{ 7,8,9 \right\}$

$\therefore A\cap B=\varnothing $

v. $A=\left\{ 1,2,3 \right\},B=\varnothing $

Ans-

Given that,

$A=\left\{ 1,2,3 \right\},B=\varnothing $

To find the intersection of two sets

The intersection of sets A and B is the set of all elements which are common to both A and B.

$A\bigcap B=\left\{ 1,2,3 \right\}\cap \varnothing $

$\therefore A\cap B=\varnothing $


6. If $A=\left\{ 3,5,7,9,11 \right\},B=\left\{ 7,9,11,13 \right\},C=\left\{ 11,13,15 \right\}$ and $D=\left\{ 15,17 \right\}$; find 

i. $A\cap B$

Ans-

Given that,

$A=\left\{ 3,5,7,9,11 \right\},B=\left\{ 7,9,11,13 \right\},C=\left\{ 11,13,15 \right\},D=\left\{ 15,17 \right\}$

To find,

$A\cap B$

The intersection of sets A and B is the set of all elements which are common to both A and B.

$A\cap B=\left\{ 3,5,7,9,11 \right\}\cap \left\{ 7,9,11,13 \right\}$

$\therefore A\cap B=\left\{ 7,9,11 \right\}$

ii. $B\cap C$

Ans-

Given that,

$A=\left\{ 3,5,7,9,11 \right\},B=\left\{ 7,9,11,13 \right\},C=\left\{ 11,13,15 \right\},D=\left\{ 15,17 \right\}$

To find,

$B\cap C$

The intersection of sets A and B is the set of all elements which are common to both A and B.

$B\cap C=\left\{ 7,9,11,13 \right\}\cap \left\{ 11,13,15 \right\}$

$\therefore B\cap C=\left\{ 11,13 \right\}$


iii. $A\cap C\cap D$

Ans-

Given that,

$A=\left\{ 3,5,7,9,11 \right\},B=\left\{ 7,9,11,13 \right\},C=\left\{ 11,13,15 \right\},D=\left\{ 15,17 \right\}$

To find,

$A\cap C\cap D$

The intersection of sets A and B is the set of all elements which are common to both A and B.

$A\cap C\cap D=\left\{ 3,5,7,9,11 \right\}\cap \left\{ 11,13,15 \right\}\cap \left\{ 15,17 \right\}$

$\therefore A\cap C\cap D=\varnothing $


iv. $A\cap C$

Ans-

Given that,

$A=\left\{ 3,5,7,9,11 \right\},B=\left\{ 7,9,11,13 \right\},C=\left\{ 11,13,15 \right\},D=\left\{ 15,17 \right\}$

To find,

$A\cap C$

The intersection of sets A and B is the set of all elements which are common to both A and B.

$A\cap C=\left\{ 3,5,7,9,11 \right\}\cap \left\{ 11,13,15 \right\}$

$\therefore A\cap C=\left\{ 11 \right\}$


v. $B\cap D$

Ans-

Given that,

$A=\left\{ 3,5,7,9,11 \right\},B=\left\{ 7,9,11,13 \right\},C=\left\{ 11,13,15 \right\},D=\left\{ 15,17 \right\}$

To find,

$B\cap D$

The intersection of sets A and B is the set of all elements which are common to both A and B.

$B\cap D=\left\{ 7,9,11,13 \right\}\cap \left\{ 15,17 \right\}$

$\therefore B\cap D=\varnothing $


vi. $A\cap \left( B\bigcup C \right)$

Ans-

Given that,

$A=\left\{ 3,5,7,9,11 \right\},B=\left\{ 7,9,11,13 \right\},C=\left\{ 11,13,15 \right\},D=\left\{ 15,17 \right\}$

To find,

$A\cap \left( B\cup C \right)$

The intersection of sets A and B is the set of all elements which are common to both A and B.

$A\cap \left( B\cup C \right)=\left( A\cap B \right)\cup \left( A\cap C \right)$

$A\cap B=\left\{ 3,5,7,9,11 \right\}\cap \left\{ 7,9,11,13 \right\}$

$A\cap B=\left\{ 7,9,11 \right\}$

$A\cap D=\left\{ 11 \right\}$

$A\cap \left( B\cup C \right)=\left\{ 7,9,11 \right\}\cup \left\{ 11 \right\}$

$=\left\{ 11 \right\}$

$\therefore A\cap \left( B\cup C \right)=\left\{ 11 \right\}$

vii. $A\cap D$

Ans-

Given that,

$A=\left\{ 3,5,7,9,11 \right\},B=\left\{ 7,9,11,13 \right\},C=\left\{ 11,13,15 \right\},D=\left\{ 15,17 \right\}$

To find, $A\cap D$

The intersection of sets A and B is the set of all elements which are common to both A and B.

$A\cap D=\left\{ 3,5,7,9,11 \right\}\cap \left\{ 15,17 \right\}$

$\therefore A\cap D=\varnothing $


viii. $A\cap \left( B\cup D \right)$

Ans-

Given that,

$A=\left\{ 3,5,7,9,11 \right\},B=\left\{ 7,9,11,13 \right\},C=\left\{ 11,13,15 \right\},D=\left\{ 15,17 \right\}$

To find,

$A\cap \left( B\cup D \right)$

The intersection of sets A and B is the set of all elements which are common to both A and B.

$A\cap \left( B\cup D \right)=\left( A\cap B \right)\cup \left( A\cap D \right)$

$A\cap B=\left\{ 3,5,7,9,11 \right\}\cap \left\{ 7,9,11,13 \right\}$

$A\cap D=\varnothing $

$\therefore A\cap \left( B\cup D \right)=\left\{ 7,9,11 \right\}\cup \varnothing $

$=\left\{ 7,9,11 \right\}$

ix. $\left( A\cap B \right)\cap \left( B\cup C \right)$

Ans-

Given that,

$A=\left\{ 3,5,7,9,11 \right\},B=\left\{ 7,9,11,13 \right\},C=\left\{ 11,13,15 \right\},D=\left\{ 15,17 \right\}$

To find,

$\left( A\cap B \right)\cap \left( B\cup C \right)$

The intersection of sets A and B is the set of all elements which are common to both A and B.

$A\cap B=\left\{ 3,5,7,9,11 \right\}\cap \left\{ 7,9,11,13 \right\}$

$A\cap B=\left\{ 7,9,11 \right\}$

$B\cup C=\left\{ 7,9,11,13 \right\}\cup \left\{ 11,13,15 \right\}$

$=\left\{ 7,9,11,13,15 \right\}$

$\left( A\cap B \right)\cap \left( B\cup C \right)=\left\{ 7,9,11 \right\}\cap \left\{ 7,9,11,13,15 \right\}$

$=\left\{ 7,9,11 \right\}$


x. $\left( A\cup D \right)\cap \left( B\cup C \right)$

Ans-

Given that,

$A=\left\{ 3,5,7,9,11 \right\},B=\left\{ 7,9,11,13 \right\},C=\left\{ 11,13,15 \right\},D=\left\{ 15,17 \right\}$

To find,

$\left( A\cup D \right)\cap \left( B\cup C \right)$

The intersection of sets A and B is the set of all elements which are common to both A and B.

$A\cap D=\left\{ 3,5,7,9,11 \right\}\cap \left\{ 15,17 \right\}$

$A\cap D=\left\{ 3,5,7,9,11,15,17 \right\}$

$B\cup C=\left\{ 7,9,11,13 \right\}\cup \left\{ 11,13,15 \right\}$

$=\left\{ 7,9,11,13,15 \right\}$

$\left( A\cup D \right)\cap \left( B\cup C \right)=\left\{ 3,5,7,9,11,15,17 \right\}\cap \left\{ 7,9,11,13,15 \right\}$

$\therefore \left( A\cup D \right)\cap \left( B\cup C \right)=\left\{ 7,9,11,15 \right\}$


7. If $A=\left\{ x:x\text{ is a natural number} \right\},B=\left\{ x:x\text{ is an even natural number} \right\}$  $C=\left\{ x:x\text{ is an odd natural number} \right\},D=\left\{ x:x\text{ is a prime number} \right\}$, find

i. $A\cap B$

Ans-

Given that,

$A=\left\{ x:x\text{ is a natural number} \right\}=\left\{ 1,2,3,4,... \right\}$

$B=\left\{ x:x\text{ is an even natural number} \right\}=\left\{ 2,4,6,8... \right\}$

$C=\left\{ x:x\text{ is an odd natural number} \right\}=\left\{ 1,3,5,7,... \right\}$

$D=\left\{ x:x\text{ is a prime number} \right\}=\left\{ 2,3,5,7,... \right\}$

To find, $A\cap B$

The intersection of sets A and B is the set of all elements which are common to both A and B.

$A\cap B=\left\{ 1,2,3,4,... \right\}\cap \left\{ 2,4,6,8... \right\}$

$\therefore A\cap B=B$  $=\left\{ x:x\text{ is an even natural number} \right\}$


ii. $A\cap C$

Ans-

Given that,

$A=\left\{ x:x\text{ is a natural number} \right\}=\left\{ 1,2,3,4,... \right\}$

$B=\left\{ x:x\text{ is an even natural number} \right\}=\left\{ 2,4,6,8... \right\}$

$C=\left\{ x:x\text{ is an odd natural number} \right\}=\left\{ 1,3,5,7,... \right\}$

$D=\left\{ x:x\text{ is a prime number} \right\}=\left\{ 2,3,5,7,... \right\}$

To find, $A\cap C$

The intersection of sets A and B is the set of all elements which are common to both A and B.

$A\cap C=\left\{ 1,2,3,4,... \right\}\cap \left\{ 1,3,5,7... \right\}$

$\therefore A\cap C=C$  $\left\{ x:x\text{ is an odd natural number} \right\}$


iii. $A\cap D$

Ans-

Given that,

$A=\left\{ x:x\text{ is a natural number} \right\}=\left\{ 1,2,3,4,... \right\}$

$B=\left\{ x:x\text{ is an even natural number} \right\}=\left\{ 2,4,6,8... \right\}$

$C=\left\{ x:x\text{ is an odd natural number} \right\}=\left\{ 1,3,5,7,... \right\}$

$D=\left\{ x:x\text{ is a prime number} \right\}=\left\{ 2,3,5,7,... \right\}$

To find, $A\cap D$

The intersection of sets A and B is the set of all elements which are common to both A and B.

$A\cap D=\left\{ 1,2,3,4,... \right\}\cap \left\{ 2,3,5,7,... \right\}$

$\therefore A\cap D=D$  $\left\{ x:x\text{ is a prime number} \right\}$


iv. $B\cap C$

Ans-

Given that,

$A=\left\{ x:x\text{ is a natural number} \right\}=\left\{ 1,2,3,4,... \right\}$

$B=\left\{ x:x\text{ is an even natural number} \right\}=\left\{ 2,4,6,8... \right\}$

$C=\left\{ x:x\text{ is an odd natural number} \right\}=\left\{ 1,3,5,7,... \right\}$

$D=\left\{ x:x\text{ is a prime number} \right\}=\left\{ 2,3,5,7,... \right\}$

To find, $B\cap C$

The intersection of sets A and B is the set of all elements which are common to both A and B.

$B\cap C=\left\{ 2,4,6,8... \right\}\cap \left\{ 1,3,5,7... \right\}$

$\therefore B\cap C=\varnothing $


v. $B\cap D$

Ans-

Given that,

$A=\left\{ x:x\text{ is a natural number} \right\}=\left\{ 1,2,3,4,... \right\}$

$B=\left\{ x:x\text{ is an even natural number} \right\}=\left\{ 2,4,6,8... \right\}$

$C=\left\{ x:x\text{ is an odd natural number} \right\}=\left\{ 1,3,5,7,... \right\}$

$D=\left\{ x:x\text{ is a prime number} \right\}=\left\{ 2,3,5,7,... \right\}$

To find, $B\cap D$

The intersection of sets A and B is the set of all elements which are common to both A and B.

$B\cap D=\left\{ 2,4,6,8,... \right\}\cap \left\{ 2,3,5,7,... \right\}$

$\therefore A\cap D=\left\{ 2 \right\}$


vi. $C\cap D$

Ans-

Given that,

$A=\left\{ x:x\text{ is a natural number} \right\}=\left\{ 1,2,3,4,... \right\}$

$B=\left\{ x:x\text{ is an even natural number} \right\}=\left\{ 2,4,6,8... \right\}$

$C=\left\{ x:x\text{ is an odd natural number} \right\}=\left\{ 1,3,5,7,... \right\}$

$D=\left\{ x:x\text{ is a prime number} \right\}=\left\{ 2,3,5,7,... \right\}$

To find, $C\cap D$

The intersection of sets A and B is the set of all elements which are common to both A and B.

$C\cap D=\left\{ 1,3,5,7,... \right\}\cap \left\{ 2,3,5,7,... \right\}$

$\therefore C\cap D=\left\{ x:x\text{ is a odd prime number} \right\}$

8. Which of the following pairs of sets are disjoint

i. $\left\{ 1,2,3,4 \right\}$ and $\left\{ x:x\text{ is a antural number and 4}\le \text{x}\le \text{6} \right\}$

Ans-

Given that,

$\left\{ 1,2,3,4 \right\}$ and 

$\left\{ x:x\text{ is a natural number and 4}\le \text{x}\le \text{6} \right\}=\left\{ 4,5,6 \right\}$

To find if the given sets are disjoint

The difference between sets A and B in this order is the set of elements that belong to A but not to B.

$\left\{ 1,2,3,4 \right\}\cap \left\{ 4,5,6 \right\}=\left\{ 4 \right\}$

Thus the element exists.

$\therefore $The given pair of sets is not a disjoint set

ii. $\left\{ a,e,i,o,u \right\}$ and $\left\{ c,d,e,f \right\}$

Ans-

Given that,

$\left\{ a,e,i,o,u \right\}$ and $\left\{ c,d,e,f \right\}$

To find if the given sets are disjoint

The difference of the sets A and B in this order is the set of elements which belong to A but not to B.

$\left\{ a,e,i,o,u \right\}\cap \left\{ c,d,e,f \right\}=\left\{ e \right\}$

Thus the element exists.

$\therefore $The given pair of sets is not a disjoint set

ii. $\left\{ x:x\text{ is an even integer} \right\}$ and $\left\{ x:x\text{ is an odd integer} \right\}$

Ans-

Given that,

$\left\{ x:x\text{ is an even integer} \right\}$ and 

$\left\{ x:x\text{ is an odd integer} \right\}$

To find if the given sets are disjoint

The difference between sets A and B in this order is the set of elements that belong to A but not to B.

$\left\{ x:x\text{ is an even integer} \right\}\cap $  $\left\{ x:x\text{ is an odd integer} \right\}=\varnothing $

Thus the element does not exist.

$\therefore $The given pair of sets is a disjoint set


9. If $A=\left\{ 3,6,9,12,15,18,21 \right\},B=\left\{ 4,8,12,16,20 \right\}$,$C=\left\{ 2,4,6,8,10,12,14,16 \right\},D=\left\{ 5,10,15,20 \right\}$

i. $A-B$

Ans-

Given that,

$A=\left\{ 3,6,9,12,15,18,21 \right\},B=\left\{ 4,8,12,16,20 \right\}$  $C=\left\{ 2,4,6,8,10,12,14,16 \right\},D=\left\{ 5,10,15,20 \right\}$

To find, $A-B$

The difference of the sets A and B in this order is the set of elements which belong to A but not to B.

$A-B=\left\{ 3,6,9,12,15,18,21 \right\}-\left\{ 4,8,12,16,20 \right\}$

$\therefore A-B=\left\{ 3,6,9,15,18,21 \right\}$


ii. $A-C$

Ans-

Given that,

$A=\left\{ 3,6,9,12,15,18,21 \right\},B=\left\{ 4,8,12,16,20 \right\}$  $C=\left\{ 2,4,6,8,10,12,14,16 \right\},D=\left\{ 5,10,15,20 \right\}$

To find, $A-C$

The difference of the sets A and B in this order is the set of elements which belong to A but not to B.

$A-C=\left\{ 3,6,9,12,15,18,21 \right\}-\left\{ 2,4,6,8,10,12,14,16 \right\}$

$\therefore A-C=\left\{ 3,9,15,18,21 \right\}$


iii. $A-D$

Ans-

Given that,

$A=\left\{ 3,6,9,12,15,18,21 \right\},B=\left\{ 4,8,12,16,20 \right\}$  $C=\left\{ 2,4,6,8,10,12,14,16 \right\},D=\left\{ 5,10,15,20 \right\}$

To find, $A-D$

The difference of the sets A and B in this order is the set of elements which belong to A but not to B.

$A-D=\left\{ 3,6,9,12,15,18,21 \right\}-\left\{ 5,10,15,20 \right\}$

$\therefore A-D=\left\{ 3,6,9,15,18,21 \right\}$


iv. $B-A$

Ans-

Given that,

$A=\left\{ 3,6,9,12,15,18,21 \right\},B=\left\{ 4,8,12,16,20 \right\}$  $C=\left\{ 2,4,6,8,10,12,14,16 \right\},D=\left\{ 5,10,15,20 \right\}$

To find, $B-A$

The difference of the sets A and B in this order is the set of elements which belong to A but not to B.

$B-A=\left\{ 4,8,12,16,20 \right\}-\left\{ 3,6,9,12,15,18,21 \right\}$

$\therefore B-A=\left\{ 4,8,16,20 \right\}$


v. $C-A$

Ans-

Given that,

$A=\left\{ 3,6,9,12,15,18,21 \right\},B=\left\{ 4,8,12,16,20 \right\}$  $C=\left\{ 2,4,6,8,10,12,14,16 \right\},D=\left\{ 5,10,15,20 \right\}$

To find,$C-A$

The difference of the sets A and B in this order is the set of elements which belong to A but not to B.

$C-A=\left\{ 2,4,6,8,10,12,14,16 \right\}-\left\{ 3,6,9,12,15,18,21 \right\}$

$\therefore C-A=\left\{ 2,4,8,10,14,16 \right\}$


vi. $D-A$

Ans-

Given that,

$A=\left\{ 3,6,9,12,15,18,21 \right\},B=\left\{ 4,8,12,16,20 \right\}$  $C=\left\{ 2,4,6,8,10,12,14,16 \right\},D=\left\{ 5,10,15,20 \right\}$

To find,$D-A$

The difference of the sets A and B in this order is the set of elements which belong to A but not to B.

$D-A=\left\{ 5,10,15,20 \right\}-\left\{ 3,6,9,12,15,18,21 \right\}$

$\therefore D-A=\left\{ 5,10,20 \right\}$


vii. $B-C$

Ans-

Given that,

$A=\left\{ 3,6,9,12,15,18,21 \right\},B=\left\{ 4,8,12,16,20 \right\}$  $C=\left\{ 2,4,6,8,10,12,14,16 \right\},D=\left\{ 5,10,15,20 \right\}$

To find, $B-C$

The difference of the sets A and B in this order is the set of elements which belong to A but not to B.

$B-C=\left\{ 4,8,12,16,20 \right\}-\left\{ 2,4,6,8,10,12,14,16 \right\}$

$\therefore B-C=\left\{ 20 \right\}$


viii. $B-D$

Ans-

Given that,

$A=\left\{ 3,6,9,12,15,18,21 \right\},B=\left\{ 4,8,12,16,20 \right\}$  $C=\left\{ 2,4,6,8,10,12,14,16 \right\},D=\left\{ 5,10,15,20 \right\}$

To find, $B-D$

The difference of the sets A and B in this order is the set of elements which belong to A but not to B.

$B-D=\left\{ 4,8,12,16,20 \right\}-\left\{ 5,10,15,20 \right\}$

$\therefore B-D=\left\{ 4,8,12,16 \right\}$


ix. $C-B$

Ans-

Given that,

$A=\left\{ 3,6,9,12,15,18,21 \right\},B=\left\{ 4,8,12,16,20 \right\}$  $C=\left\{ 2,4,6,8,10,12,14,16 \right\},D=\left\{ 5,10,15,20 \right\}$

To find, $C-B$

The difference of the sets A and B in this order is the set of elements which belong to A but not to B.

$C-B=\left\{ 2,4,6,8,10,12,14,16 \right\}-\left\{ 4,8,12,16,20 \right\}$

$\therefore C-B=\left\{ 2,6,10,14 \right\}$


x. $D-B$

Ans-

Given that,

$A=\left\{ 3,6,9,12,15,18,21 \right\},B=\left\{ 4,8,12,16,20 \right\}$  $C=\left\{ 2,4,6,8,10,12,14,16 \right\},D=\left\{ 5,10,15,20 \right\}$

To find, $D-B$

The difference of the sets A and B in this order is the set of elements which belong to A but not to B.

$D-B=\left\{ 5,10,15,20 \right\}-\left\{ 4,8,12,16,20 \right\}$

$\therefore D-B=\left\{ 5,10,15 \right\}$


xi. $C-D$

Ans-

Given that,

$A=\left\{ 3,6,9,12,15,18,21 \right\},B=\left\{ 4,8,12,16,20 \right\}$  $C=\left\{ 2,4,6,8,10,12,14,16 \right\},D=\left\{ 5,10,15,20 \right\}$

To find, $C-D$

The difference of the sets A and B in this order is the set of elements which belong to A but not to B.

$C-D=\left\{ 2,4,6,8,10,12,14,16 \right\}-\left\{ 5,10,15,20 \right\}$

$\therefore C-D=\left\{ 2,4,6,8,12,14,16 \right\}$


xii. $D-C$

Ans-

Given that,

$A=\left\{ 3,6,9,12,15,18,21 \right\},B=\left\{ 4,8,12,16,20 \right\}$  $C=\left\{ 2,4,6,8,10,12,14,16 \right\},D=\left\{ 5,10,15,20 \right\}$

To find, $D-C$

The difference of the sets A and B in this order is the set of elements which belong to A but not to B.

$D-C=\left\{ 5,10,15,20 \right\}-\left\{ 2,4,6,8,10,12,14,16 \right\}$

$\therefore D-C=\left\{ 5,10,15 \right\}$


10. If $X=\left\{ a,b,c,d \right\},Y=\left\{ f,b,d,g \right\}$, find

i. $X-Y$

Ans-

Given that,

$X=\left\{ a,b,c,d \right\},Y=\left\{ f,b,d,g \right\}$

To find,

$X-Y$

The difference of the sets A and B in this order is the set of elements which belong to A but not to B.

$X-Y=\left\{ a,b,c,d \right\}-\left\{ f,b,d,g \right\}$

$\therefore X-Y=\left\{ a,c \right\}$


ii. $Y-X$

Ans-

Given that,

$X=\left\{ a,b,c,d \right\},Y=\left\{ f,b,d,g \right\}$

To find, $Y-X$

The difference of the sets A and B in this order is the set of elements which belong to A but not to B.

$Y-X=\left\{ f,b,d,g \right\}-\left\{ a,b,c,d \right\}$

$\therefore Y-X=\left\{ f,g \right\}$


iii. $X\cap Y$

Ans-

Given that,

$X=\left\{ a,b,c,d \right\},Y=\left\{ f,b,d,g \right\}$

To find,

$X\cap Y$

The difference of the sets A and B in this order is the set of elements which belong to A but not to B.

$X\cap Y=\left\{ a,b,c,d \right\}\cap \left\{ f,b,d,g \right\}$

$\therefore X\cap Y=\left\{ b,d \right\}$


11. If R is the real numbers and Q is the set of rational numbers, then what is $R-Q$?

Ans-

Given that,

R is the real numbers

Q is the set of rational numbers

To find, $R-Q$

The difference of the sets A and B in this order is the set of elements which belong to A but not to B.

$\therefore R-Q$ is the set of irrational numbers.


12. State whether each of the following statements is true or false. Justify your answer.

i. $\left\{ 2,3,4,5 \right\}$ and $\left\{ 3,6 \right\}$ are disjoint sets

Ans-

Given that,

$\left\{ 2,3,4,5 \right\},\left\{ 3,6 \right\}$

To state whether the given statement is true

The difference of the sets A and B in this order is the set of elements which belong to A but not to B.

$\left\{ 2,3,4,5 \right\}\cap \left\{ 3,6 \right\}=\left\{ 3 \right\}$

$\therefore $The given statement is false.


ii. $\left\{ a,e,i,o,u \right\}$ and $\left\{ a,b,c,d \right\}$ are disjoint sets

Ans-

Given that,

$\left\{ a,e,i,o,u \right\},\left\{ a,b,c,d \right\}$

To state whether the given statement is true

The difference of the sets A and B in this order is the set of elements which belong to A but not to B.

$\left\{ a,e,i,o,u \right\}\cap \left\{ a,b,c,d \right\}=\left\{ a \right\}$

$\therefore $The given statement is false.


iii. $\left\{ 2,6,10,14 \right\}$ and $\left\{ 3,7,11,15 \right\}$ are disjoint sets

Ans-

Given that,

$\left\{ 2,6,10,14 \right\},\left\{ 3,7,11,15 \right\}$

To state whether the given statement is true

The difference of the sets A and B in this order is the set of elements which belong to A but not to B.

$\left\{ 2,6,10,14 \right\}\cap \left\{ 3,7,11,15 \right\}=\varnothing $

$\therefore $The given statement is true.


iv. $\left\{ 2,6,10 \right\}$ and $\left\{ 3,7,11 \right\}$ are disjoint sets

Ans-

Given that,

$\left\{ 2,6,10 \right\},\left\{ 3,7,11 \right\}$

To state whether the given statement is true

$\left\{ 2,6,10 \right\}\cap \left\{ 3,7,11 \right\}=\varnothing $

$\therefore $The given statement is true.

Exercise (1.5)

1. Let $U=\left\{ 1,2,3,4,5,6,7,8,9 \right\},A=\left\{ 1,2,3,4 \right\},B=\left\{ 2,4,6,8 \right\}$ and $C=\left\{ 3,4,5,6 \right\}$, find 

i. $A'$

Ans-

Given that,

$U=\left\{ 1,2,3,4,5,6,7,8,9 \right\},A=\left\{ 1,2,3,4 \right\},B=\left\{ 2,4,6,8 \right\}$, $C=\left\{ 3,4,5,6 \right\}$

To find, $A'$

The complement of set A is the set of all elements of U which are not the elements of A.

$A'=U-A$

$=\left\{ 1,2,3,4,5,6,7,8,9 \right\}-\left\{ 1,2,3,4 \right\}$

$=\left\{ 5,6,7,8,9 \right\}$

$\therefore A'=\left\{ 5,6,7,8,9 \right\}$


ii. $B'$

Ans-

Given that,

$U=\left\{ 1,2,3,4,5,6,7,8,9 \right\},A=\left\{ 1,2,3,4 \right\},B=\left\{ 2,4,6,8 \right\}$, $C=\left\{ 3,4,5,6 \right\}$

To find, $B'$

The complement of set A is the set of all elements of U which are not the elements of A.

$B'=U-B$

$=\left\{ 1,2,3,4,5,6,7,8,9 \right\}-\left\{ 2,4,6,8 \right\}$

$=\left\{ 1,3,5,7,9 \right\}$

$\therefore B'=\left\{ 1,3,5,7,9 \right\}$


iii. $\left( A\cup C \right)'$

Ans-

Given that,

$U=\left\{ 1,2,3,4,5,6,7,8,9 \right\},A=\left\{ 1,2,3,4 \right\},B=\left\{ 2,4,6,8 \right\}$, $C=\left\{ 3,4,5,6 \right\}$

To find,

$\left( A\cup C \right)'$

The complement of set A is the set of all elements of U which are not the elements of A.

$A\cup C=\left\{ 1,2,3,4,5,6 \right\}$

$\left( A\cup C \right)'=U-\left( A\cup C \right)$

$=\left\{ 1,2,3,4,5,6,7,8,9 \right\}-\left\{ 1,2,3,4,5,6 \right\}$

$=\left\{ 7,8,9 \right\}$

$\therefore \left( A\cup C \right)'=\left\{ 7,8,9 \right\}$


iv. $\left( A\cup B \right)'$

Ans-

Given that,

$U=\left\{ 1,2,3,4,5,6,7,8,9 \right\},A=\left\{ 1,2,3,4 \right\},B=\left\{ 2,4,6,8 \right\}$, $C=\left\{ 3,4,5,6 \right\}$

To find,

$\left( A\cup B \right)'$

The complement of set A is the set of all elements of U which are not the elements of A.

$A\cup B=\left\{ 1,2,3,4,5,6,8 \right\}$

$\left( A\cup B \right)'=U-A\cup B$

$=\left\{ 1,2,3,4,5,6,7,8,9 \right\}-\left\{ 1,2,3,4,5,6,8 \right\}$

$=\left\{ 5,7,9 \right\}$

$\therefore \left( A\cup B \right)'=\left\{ 5,7,9 \right\}$


v. $\left( A' \right)'$

Ans-

Given that,

$U=\left\{ 1,2,3,4,5,6,7,8,9 \right\},A=\left\{ 1,2,3,4 \right\},B=\left\{ 2,4,6,8 \right\}$, $C=\left\{ 3,4,5,6 \right\}$

To find,

$\left( A' \right)'$

The complement of set A is the set of all elements of U which are not the elements of A.

$\left( A' \right)'=A$

$=\left\{ 1,2,3,4 \right\}$

$\therefore \left( A' \right)'=\left\{ 1,2,3,4 \right\}$


vi. $\left( B-C \right)'$

Ans-

Given that,

$U=\left\{ 1,2,3,4,5,6,7,8,9 \right\},A=\left\{ 1,2,3,4 \right\},B=\left\{ 2,4,6,8 \right\}$, $C=\left\{ 3,4,5,6 \right\}$

To find,

$\left( B-C \right)'$

The complement of set A is the set of all elements of U which are not the elements of A.

$B-C=\left\{ 2,8 \right\}$

$\left( B-C \right)'=U-\left( B-C \right)$

$=\left\{ 1,2,3,4,5,6,7,8,9 \right\}-\left\{ 2,8 \right\}$

$=\left\{ 1,3,4,5,6,7,9 \right\}$

$\therefore \left( B-C \right)'=\left\{ 1,3,4,5,6,7,9 \right\}$


2. If $U=\left\{ a,b,c,d,e,f,g,h \right\}$, then find the complements of the following sets:

i. $A=\left\{ a,b,c \right\}$

Ans-

Given that,

$U=\left\{ a,b,c,d,e,f,g,h \right\}$

$A=\left\{ a,b,c \right\}$

To find the complement of A

The complement of set A is the set of all elements of U which are not the elements of A.

$A'=U-A$

$=\left\{ a,b,c,d,e,f,g,h \right\}-\left\{ a,b,c \right\}$

$=\left\{ d,e,f,g,h \right\}$

$\therefore $The complement of $A$ is $A'=\left\{ d,e,f,g,h \right\}$


ii. $B=\left\{ d,e,f,g \right\}$

Ans-

Given that,

$U=\left\{ a,b,c,d,e,f,g,h \right\}$

$b=\left\{ d,e,f,g \right\}$

To find the complement of B

The complement of set A is the set of all elements of U which are not the elements of A.

$B'=U-B$

$=\left\{ a,b,c,d,e,f,g,h \right\}-\left\{ d,e,f,g \right\}$

$=\left\{ a,b,c,h \right\}$

$\therefore $The complement of $B$ is $B'=\left\{ b,e,c,h \right\}$


iii. $C=\left\{ a,c,e,g \right\}$

Ans-

Given that,

$U=\left\{ a,b,c,d,e,f,g,h \right\}$

$C=\left\{ a,c,e,g \right\}$

To find the complement of A

The complement of set A is the set of all elements of U which are not the elements of A.

$C'=U-C$

$=\left\{ a,b,c,d,e,f,g,h \right\}-\left\{ a,c,e,g \right\}$

$=\left\{ b,d,f,h \right\}$

$\therefore $The complement of $C$ is $C'=\left\{ b,d,f,h \right\}$


iv. $D=\left\{ f,g,h,a \right\}$

Ans-

Given that,

$U=\left\{ a,b,c,d,e,f,g,h \right\}$

$A=\left\{ f,g,h,a \right\}$

To find the complement of A

The complement of set A is the set of all elements of U which are not the elements of A.

$D'=U-D$

$=\left\{ a,b,c,d,e,f,g,h \right\}-\left\{ f,g,h,a \right\}$

$=\left\{ b,c,d,e \right\}$

$\therefore $The complement of $D$ is $D'=\left\{ b,c,d,e \right\}$


3. Taking the set of natural numbers as the universal set, write down the complements of the following sets:

i. $\left\{ x:x\text{ is an even natural number} \right\}$

Ans-

Given that,

The set of natural number is the universal set

To find the complement of the set of even natural number

The complement of set A is the set of all elements of U which are not the elements of A.

$\therefore \left\{ x:x\text{ is an even natural number} \right\}'=\left\{ x:x\text{ is an odd natural number} \right\}$


ii. $\left\{ x:x\text{ is an odd natural number} \right\}$

Ans-

Given that,

The set of natural numbers is the universal set

To find the complement of the set of odd natural number

The complement of set A is the set of all elements of U which are not the elements of A.

$\therefore \left\{ x:x\text{ is an odd natural number} \right\}'=\left\{ x:x\text{ is an even natural number} \right\}$


iii. $\left\{ x:x\text{ is a positive multiple of 3} \right\}$

Ans-

Given that,

The set of natural number is the universal set

To find the complement of the set of positive multiples of $3$

The complement of set A is the set of all elements of U which are not the elements of A.

$\therefore \left\{ x:x\text{ is a positive multiple of 3} \right\}'=\left\{ x:x\in N\text{ and x is not a positive multiple of 3} \right\}$


iv. $\left\{ x:x\text{ is a prime number} \right\}$

Ans-

Given that,

The set of natural number is the universal set

To find the complement of the set of prime number

The complement of set A is the set of all elements of U which are not the elements of A.

$\therefore \left\{ x:x\text{ is a prime number} \right\}'=\left\{ x:x\text{ is a positive composite number and x=1} \right\}$


v. $\left\{ x:x\text{ is a natural number divisible by 3 and 5} \right\}$

Ans-

Given that,

The set of natural number is the universal set

To find the complement of the set of natural number divisible by $3$ and $5$

The complement of set A is the set of all elements of U which are not the elements of A.

$\therefore \left\{ x:x\text{ is a number divisible by 3 and 5} \right\}'=$

$\left\{ x:x\text{ is a natural number that is not divisible by 3 or 5} \right\}$


vi. $\left\{ x:x\text{ is a perfect square} \right\}$

Ans-

Given that,

The set of natural number is the universal set

To find the complement of the set of perfect squares.

The complement of set A is the set of all elements of U which are not the elements of A.

$\therefore \left\{ x:x\text{ is a perfect squares} \right\}'=\left\{ x:x\in N\text{ and x is not a perfect square} \right\}$

vii. $\left\{ x:x\text{ is a perfect cube} \right\}$

Ans-

Given that,

The set of natural number is the universal set

To find the complement of the set of perfect cube

The complement of set A is the set of all elements of U which are not the elements of A.

$\therefore \left\{ x:x\text{ is a perfect cube} \right\}'=\left\{ x:x\in N\text{ and x is not a perfect cube} \right\}$

viii. $\left\{ x:x+5=8 \right\}$

Ans-

Given that,

The set of natural number is the universal set

To find the complement of $\left\{ x:x+5=8 \right\}$

$x+5=8$

$x=3$

The complement of set A is the set of all elements of U which are not the elements of A.

$\therefore \left\{ x:x+5=8 \right\}'=\left\{ x:x\in N\text{ and x}\ne \text{3} \right\}$

ix. $\left\{ x:2x+5=9 \right\}$

Ans-

Given that,

The set of natural number is the universal set

To find the complement of the 

$\left\{ x:2x+5=9 \right\}$

The complement of set A is the set of all elements of U which are not the elements of A.

$2x+5=9$

$2x=4$

$x=2$

$\therefore \left\{ x:2x+5=9 \right\}'=\left\{ x:x\in N\text{ and x}\ne \text{2 } \right\}$

x. $\left\{ x:x\ge 7 \right\}$

Ans-

Given that,

The set of natural number is the universal set

To find the complement of 

$\left\{ x:x\ge 7 \right\}$

The complement of set A is the set of all elements of U which are not the elements of A.

$\therefore \left\{ x:x\ge 7 \right\}'=\left\{ x:x\in N\text{ and x7} \right\}$


xi. $\left\{ x:x\in N\text{ and 2x+110} \right\}$

Ans-

Given that,

The set of natural number is the universal set

To find the complement of the 

$\left\{ x:x\in N\text{ and 2x+110} \right\}$

The complement of set A is the set of all elements of U which are not the elements of A.

$2x+1>10$

$2x>9$

$x>\frac{9}{2}$

$\therefore \left\{ x:x\in N\text{ and 2x+110} \right\}'$  $=\left\{ x:x\in N\text{ and x}\le \frac{9}{2} \right\}$


4. If $U=\left\{ 1,2,3,4,5,6,7,8,9 \right\},A=\left\{ 2,4,6,8 \right\}$ and $B=\left\{ 2,3,5,7 \right\}$. Verify that,

i. $\left( A\cup B \right)'=A'\cap B'$

Ans-

Given that.

$U=\left\{ 1,2,3,4,5,6,7,8,9 \right\}$

$A=\left\{ 2,4,6,8 \right\}$

$B=\left\{ 2,3,5,7 \right\}$

To prove that $\left( A\cup B \right)'=A'\cap B'$

$A\cup B=\left\{ 2,4,6,8 \right\}\cup \left\{ 2,3,5,7 \right\}$

$=\left\{ 2,3,4,5,6,7,8 \right\}$

$\left( A\cup B \right)'=U=A\cup B$

$=\left\{ 1,9 \right\}$

$A'=U-A$

$=\left\{ 1,3,5,7,9 \right\}$

$B'=U-B$

$=\left\{ 1,4,6,8,9 \right\}$

$A'\cap B'=\left\{ 1,3,5,7,9 \right\}\cap \left\{ 1,4,6,8,9 \right\}$

$=\left\{ 1,9 \right\}$

Hence it has been proved that $\left( A\cup B \right)'=A'\cap B'$


ii. $\left( A\cap B \right)'=A'\cup B'$

Ans-

Given that.

$U=\left\{ 1,2,3,4,5,6,7,8,9 \right\}$

$A=\left\{ 2,4,6,8 \right\}$

$B=\left\{ 2,3,5,7 \right\}$

To prove that $\left( A\cap B \right)'=A'\cup B'$

$A\cap B=\left\{ 2,4,6,8 \right\}\cap \left\{ 2,3,5,7 \right\}$

$=\left\{ 2 \right\}$

$\left( A\cap B \right)'=U-A\cap B$

$=\left\{ 1,3,4,5,6,7,8,9 \right\}$

$A'=U-A$

$=\left\{ 1,3,5,7,9 \right\}$

$B'=U-B$

$=\left\{ 1,4,6,8,9 \right\}$

$A'\cup B'=\left\{ 1,3,5,7,9 \right\}\cup \left\{ 1,4,6,8,9 \right\}$

$=\left\{ 1,3,4,5,6,7,8,9 \right\}$

Hence it has been proved that $\left( A\cap B \right)'=A'\cup B'$


5. Draw appropriate Venn diagrams for each of the following:

i. $\left( A\cup B \right)'$

Ans-

To draw the Venn diagram for $\left( A\cup B \right)'$

(Image Will Be Updated Soon)


ii. $A'\cap B'$

Ans-

To draw the Venn diagram for $A'\cap B'$

(Image Will Be Updated Soon)


iii. $\left( A\cap B \right)'$

Ans-

To draw the Venn diagram for $\left( A\cap B \right)'$

(Image Will Be Updated Soon


iv. $A'\cup B'$

Ans-

To draw the Venn diagram for $A'\cup B'$

(Image Will Be Updated Soon)


6. Let $U$ be the set of all triangles in a plane. If $A$ is the set of all triangles with at least one angle different from $60{}^\circ $, what is $A'$?

Ans-

Given that,

$U$ is the set of all triangles in the plane

$A=$Set of triangles different form $60{}^\circ $

To find $A'$

The complement of set A is the set of all elements of U which are not the elements of A.

$A'=U-A$

$=$Set of all equilateral triangles

$\therefore A'$ is the set of all equilateral triangles


7. Fill in the blanks to make each of the following a true statement:

i. $A\cup A'=...$

Ans-

To fill the blanks given in the statement

The union of the set and its complement is the universal set

$\therefore A\cup A'=U$

ii. $\varnothing '\cap A=...$

Ans-

To fill the blanks given in the statement

We know that,

$\varnothing '\cap A=U\cap A=A$

$\therefore \varnothing '\cap A=A$


iii. $A\cap A'=...$

Ans-

To fill the blanks given in the statement

The intersection of the set and its complement is an empty set.

$\therefore A\cap A'=\varnothing $


iv. $U'\cap A=...$

Ans-

To fill the blanks given in the statement

We know that,

$\varnothing \cap A=U'\cap A=\varnothing $

$\therefore U'\cap A=\varnothing $


Exercise (1.6)

1. If $X$ and $Y$ are two sets such that $n\left( X \right)=17,n\left( Y \right)=23$ and $n\left( X\cup Y \right)=38$, find $n\left( X\cap Y \right)$

Ans-

Given that,

$n\left( X \right)=17$

$n\left( Y \right)=23$

$n\left( X\cup Y \right)=38$

To find,

$n\left( X\cap Y \right)$

We know that,

$n\left( X\cup Y \right)=n\left( X \right)+n\left( Y \right)-n\left( X\cap Y \right)$

$n\left( X\cap Y \right)=\left( 17+23 \right)-38$

$\therefore n\left( X\cap Y \right)=2$


2. If $X$ and $Y$are two sets such that $X\cup Y$has$18$ elements, $X$ has $8$ elements and $Y$ has $15$ elements: how many elements does $X\cap Y$ have?

Ans-

Given that,

$n\left( X \right)=8$

$n\left( Y \right)=15$

$n\left( X\cup Y \right)=18$

To find,

$n\left( X\cap Y \right)$

We know that,

$n\left( X\cup Y \right)=n\left( X \right)+n\left( Y \right)-n\left( X\cap Y \right)$

$n\left( X\cap Y \right)=\left( 8+15 \right)-18$

$\therefore n\left( X\cap Y \right)=5$


3. In a group of $400$ people, $250$can speak Hindi and $200$ can speak English. How many people can speak both Hindi and English?

Ans-

Let the set of people who are speaking Hindi are denoted by $H$ and the set of people who are speaking English be denoted by $E$

Given that,

Given that,

$n\left( H \right)=250$

$n\left( E \right)=200$

$n\left( H\cup E \right)=400$

To find,

$n\left( H\cap E \right)$

We know that,

$n\left( H\cup E \right)=n\left( H \right)+n\left( E \right)-n\left( H\cap E \right)$

$n\left( H\cap E \right)=\left( 250+200 \right)-400$

$n\left( H\cap E \right)=50$

$\therefore 50$ people can speak both English and Hindi.


4. If $S$ and $T$ are two sets such that $S$ has $21$ elements, $T$ has $32$ elements, and $S\cap T$ has $11$ elements, how many elements does $S\cup T$ have?

Ans-

Given that,

$n\left( S \right)=21$

$n\left( T \right)=32$

$n\left( S\cap T \right)=11$

To find,

$n\left( S\cup T \right)$

We know that,

$n\left( S\cup T \right)=n\left( S \right)+n\left( T \right)-n\left( S\cap T \right)$

$n\left( S\cup T \right)=21+32-11$

$\therefore n\left( S\cup T \right)=42$

$\therefore S\cup T$ have $42$ elements


5. If $X$ and $Y$are two sets such that $X$ has$40$ elements and $X\cup Y$ has $60$ elements, $X\cap Y$ have $10$ elements, how many elements does $Y$ have?

Ans-

Given that,

$n\left( X \right)=40$

$n\left( X\cup Y \right)=60$

$n\left( X\cap Y \right)=10$

To find,

$n\left( Y \right)$

We know that,

$n\left( X\cup Y \right)=n\left( X \right)+n\left( Y \right)-n\left( X\cap Y \right)$

$60=40+n\left( Y \right)-10$

$n\left( Y \right)=60-\left( 40-10 \right)$

$=30$

$\therefore n\left( Y \right)$ has $30$ elements.


6. In a group of $70$ people, $37$ like coffee, $52$ like tea and each person likes at least one of the two drinks. How many people like both coffee and tea?

 Ans-

Let the people who like coffee be denoted by $C$ and the set of people who like tea be denoted by $T$ 

Given that,

$n\left( C \right)=37$

$n\left( T \right)=52$

$n\left( C\cup T \right)=70$

To find,

$n\left( C\cap T \right)$

We know that,

$n\left( C\cup T \right)=n\left( C \right)+n\left( T \right)-n\left( C\cap T \right)$

$70=37+52-n\left( C\cap T \right)$

$n\left( C\cap T \right)=\left( 37+52 \right)-70$

$\therefore n\left( C\cap T \right)=19$

$\therefore $The number of people who like both coffee and tea are $19$ people


7. In a group of $65$ people, $40$ like cricket, $10$ like both cricket and tennis. How many like tennis only and not cricket? How many like tennis?

Ans-

Let the number of people who like cricket be denoted by $C$ and the number of people who like tennis be denoted by $T$

Given that,

$n\left( C \right)=40$

$n\left( C\cup T \right)=65$

$n\left( C\cap T \right)=10$

To find,

$n\left( T \right)$

$n\left( T-C \right)$

We know that,

$n\left( C\cup T \right)=n\left( C \right)+n\left( T \right)-n\left( C\cap T \right)$

$65=40+n\left( T \right)-10$

$n\left( T \right)=65-30$

$\therefore n\left( T \right)=35$

The number of people who like Tennis is $35$ people.

Now, $\left( T-C \right)\cup \left( T\cap C \right)=T$

And $\left( T-C \right)\cap \left( T\cap C \right)=\varnothing $

$n\left( T \right)=n\left( T-C \right)+n\left( T\cap C \right)$

$35=n\left( T-C \right)+10$

$n\left( T-C \right)=25$

$\therefore $The number of people who like only Tennis is $25$ people.


8. In a committee $50$ people speak French, $20$ people speak Spanish and $10$ people speak both Spanish and French. How many speak at least one of these two languages?

Ans-

Let the people who speak French be denoted by $F$ and the number of people who speak Spanish be denoted by $S$

Given that,

$n\left( S \right)=20$

$n\left( F \right)=50$

$n\left( S\cap F \right)=10$

To find,

$n\left( S\cup F \right)$

We know that,

$n\left( S\cup F \right)=n\left( S \right)+n\left( F \right)-n\left( S\cap F \right)$

$=20+50-10$

$=60$

$\therefore $The number of people who can speak at least one of the languages is $60$ people.


Miscellaneous Exercise 1

1. Decide among the following sets, which sets are the subsets of one and another:

$A=\left\{ x:x\in R\text{ satisfy }{{\text{x}}^{2}}-8x+12=0 \right\}$

$B=\left\{ 2,4,6 \right\}$

$C=\left\{ 2,4,6,8,... \right\},D=\left\{ 6 \right\}$

Ans-

Given that,

$A=\left\{ x:x\in R\text{ satisfy }{{\text{x}}^{2}}-8x+12=0 \right\},B=\left\{ 2,4,6 \right\}$

$C=\left\{ 2,4,6,8,... \right\},D=\left\{ 6 \right\}$

$A=\left\{ x:x\in R\text{ satisfy }{{\text{x}}^{2}}-8x+12=0 \right\}$

${{x}^{2}}-8x+12=0$

$\left( x-2 \right)\left( x-6 \right)=0$

$x=2,6$

$A=\left\{ 2,6 \right\},B=\left\{ 2,4,6 \right\},$  $C=\left\{ 2,4,6,8,... \right\},D=\left\{ 6 \right\}$

A set A is said to be a subset of B if every element of A is also an element of B

$A\subset B$ if $a\in A,a\in B$

We can observe that,

$D\subset A\subset B\subset C$

$\therefore D\subset A,B,C\text{ and A}\subset B,C\text{ and B}\subset C$


2. In each of the following statements, determine whether the statement is true or false. If it is true, prove it. If it is false, give an example.

i. If $x\in A\text{ and A}\in \text{B, then x}\in \text{B}$

Ans-

$x\in A\text{ and A}\in \text{B, then x}\in \text{B}$

To determine whether the statement is true

The given statement is false

For example,

Let $A=\left\{ 1,2 \right\},B=\left\{ 1,\left\{ 1,2 \right\},\left\{ 3 \right\} \right\}$

Now $2\in \left\{ 1,2 \right\},\left\{ 1,2 \right\}\in \left\{ 1,\left\{ 1,2 \right\},\left\{ 3 \right\} \right\}$

But $2\notin \left\{ 1,\left\{ 1,2 \right\},\left\{ 3 \right\} \right\}$

Hence the given statement is false.


ii. If $A\subset B\text{ and }B\in C,thenA\in C$

Ans-

$A\subset B\text{ and }B\in C,thenA\in C$

To determine whether the statement is true

The statement is false

For example,

Let $A=\left\{ 2 \right\},B=\left\{ 0,2 \right\},C=\left\{ 1,\left\{ 0,2 \right\},3 \right\}$

As $A\subset B,B\in C$ but $A\notin C$


iii. If $A\subset B\text{ and }B\subset C,thenA\subset C$

Ans-

$A\subset B\text{ and }B\subset C,thenA\subset C$

To determine whether the statement is true

The given statement is true.

Let $A\subset B,B\subset C$

Let $x\in A$ 

$x\in B$so $A\subset B$

$x\in C$so $B\subset C$


iv. If $A\not\subset B\text{ and }B\not\subset C,thenA\not\subset C$

Ans-

$A\not\subset B\text{ and }B\not\subset C,thenA\not\subset C$

To determine whether the statement is true

The given statement is false

Let $A=\left\{ 1,2 \right\},B=\left\{ 0,6,8 \right\},C=\left\{ 0,1,2,6,9 \right\}$

Now by the statement,

$A\not\subset B\text{ and }B\not\subset C$

But $A\subset C$


v. If $x\in A\text{ and A}\not\subset \text{B, then x}\in \text{B}$

Ans-

$x\in A\text{ and A}\not\subset \text{B, then x}\in \text{B}$

To determine whether the statement is true

The given statement is false

Let $A=\left\{ 3,5,7 \right\},B=\left\{ 3,4,6 \right\}$

Now, $5\in A$ and $A\not\subset B$

But $5\notin B$


vi. If $\text{A}\subset \text{B and x}\notin \text{B, then x}\notin \text{A}$

Ans-

$\text{A}\subset \text{B and x}\notin \text{B, then x}\notin \text{A}$

To determine whether the statement is true

The given statement is true

Let $A\subset B$ and $x\notin B$

To show that $x\notin A$

Suppose $x\in A$

Then $x\in B$ which is a contradiction

$\therefore x\notin A$


3. Let $A,B$ and $C$ be the set such that $A\cup B=A\cup C$ and $A\cap B=A\cap C$. Show that $B=C$

Ans-

To show that $B=C$

Let $x\in B$

$x\in A\cup B$ since $\left[ B\subset A\cup B \right]$

$x\in A\cup C$ since $\left[ A\cup B=A\cup C \right]$

$x\in A\text{ or }\in \text{C}$

Case I:

Also $x\in B$

$x\in A\cap B$ since $\left[ B\subset A\cap B \right]$

$x\in A\cap C$ since $\left[ A\cap B=A\cap C \right]$

$x\in A\text{ or x}\in \text{C}$

$\therefore x\in C$

$\therefore B\subset C$

Similarly, we can show that $C\subset B$

$\therefore $It has been proved that $B=C$


4. Show that the following four conditions are equivalent:

i. $A\subset B$

ii. $A-B=\varnothing $

iii. $A\cup B=B$

iv. $A\cap B=A$

Ans-

To show that the above four conditions are equivalent

First showing that,

(i)⬄(ii),

Let $A\subset B$

To show,

$A-B=\varnothing $

If possible, suppose,

$A-B\ne \varnothing $

This means that there exist $x\in A,x\notin B$, which is impossible as $A\subset B$

$\therefore A-B=\varnothing $

$A\subset B$=> $A-B=\varnothing $

Let $A-B=\varnothing $

To show that,

$A\subset B$

Let $x\in B$ since if $x\notin B,$ then $A-B\ne \varnothing $

$\therefore $  $A-B=\varnothing $ => $A\subset B$

$\therefore $(i)⬄ (iii)

Let $A\subset B$

To show $A\cup B=B$

Clearly$B\subset A\cup B$

Let $x\in A\cup B$

$x\in A$ or $x\in B$

Case I:

$x\in A$

So that $x\in B$

$\therefore A\cup B\subset B$

Case II:

$x\in B$

Then $A\cup B=B$

Conversely let $A\cup B=B$

Let $x\in A$

$x\in A\cup B$ since $\left[ A\subset A\cup B \right]$

$x\in B$ since $A\cup B=B$

$\therefore A\subset B$

Hence (i) ⬄ (iii)

Now we have to show that (i) ⬄ (iv)

Let $A\subset B$

$A\cap B\subset A$

Let $x\in A$

We have to show that $x\in A\cap B$

As $A\subset B$, $x\in B$

$x\in A\cap B$

$A\subset A\cap B$

Hence $A=A\cap B$

Let $x\in A$

$x\in A\cap B$

$x\in A$ and $x\in B$

So that $x\in B$

$\therefore A\subset B$

Hence (i) ⬄ (iv)


5. Show that if$A\subset B$, then $C-B\subset C-A$

Ans-

Given that,

$A\subset B$

To show that,

$C-B\subset C-A$

Let $x\in C-B$

$x\in C$ and $x\in B$

$x\notin A\left[ A\subset B \right]$ and $x\in C$

$x\in C-A$

$\therefore C-B\subset C-A$

Hence it has been showed that $C-B\subset C-A$


6. Assume that $P\left( A \right)=P\left( B \right)$. Show that $A=B$

Ans-

Given that,

$P\left( A \right)=P\left( B \right)$

To show that,

$A=B$

Let $x\in A$

$A\in P\left( A \right)=P\left( B \right)$

$\therefore x\in C$ for some $C\in P\left( B \right)$

Now $C\subset B$

So $x\in B$ and so $A\subset B$

Similarly $B\subset A$

$\therefore A=B$


7. Is it true for any sets $A$ and $B$, $P\left( A \right)\cup P\left( B \right)=P\left( A\cup B \right)$? Justify your answer.

Ans-

Let $A=\left\{ 0,1 \right\}$ and $B=\left\{ 1,2 \right\}$

$A\cup B=\left\{ 0,1,2 \right\}$

$P\left( A \right)=\left\{ \varnothing ,\left\{ 0 \right\},\left\{ 1 \right\},\left\{ 0,1 \right\} \right\}$

$P\left( B \right)=\left\{ \varnothing ,\left\{ 1 \right\},\left\{ 2 \right\},\left\{ 1,2 \right\} \right\}$

$P\left( A\cup B \right)=$  $\left\{ \varnothing ,\left\{ 0 \right\},\left\{ 1 \right\},\left\{ 2 \right\},\left\{ 0,1 \right\},\left\{ 1,2 \right\},\left\{ 0,2 \right\},\left\{ 0,1,2 \right\} \right\}$

$P\left( A \right)\cup P\left( B \right)=$  $\left\{ \varnothing ,\left\{ 1 \right\},\left\{ 0,1 \right\},\left\{ 2 \right\},\left\{ 1,2 \right\} \right\}$

$P\left( A \right)\ne P\left( B \right)$

$\therefore $The given statement is false


8. Show that for any sets $A$ and$B$,

$A=\left( A\cap B \right)\cup \left( A-B \right)$ and $A\cup \left( B-A \right)=A\cup B$

Ans-

To show that,

$A=\left( A\cap B \right)\cup \left( A-B \right)$

Let $x\in A$

We have to show that $A=\left( A\cap B \right)\cup \left( A-B \right)$

Case I:

$x\in A\cap B$

Then $x\in A\cap B\subset \left( A\cup B \right)\cup \left( A-B \right)$

Case II:

$x\notin A\cap B$

$x\notin A$ or $x\notin B$

$x\notin B\left[ x\notin A \right]$

$x\notin A-B\subset \left( A\cup B \right)\cup \left( A-B \right)$

$\therefore A\subset \left( A\cap B \right)\cup \left( A-B \right)$                                          ……(1)

It is clear that,

$A\cap B\subset A$ and $A-B\subset A$

$\therefore \left( A\cap B \right)\cup \left( A-B \right)\subset A$                                          ……(2)

From (1) and (2) we obtain that,

$A=\left( A\cap B \right)\cup \left( A-B \right)$

To prove that,

$A\cup \left( B-A \right)\subset A\cup B$

Let $x\in A\cup \left( B-A \right)$

$x\in A$ or ($x\in B$ and $x\notin A$)

($x\in A$ or $x\in B$) and ($x\in A$ or $x\notin A$)

$x\in A\cup B$

$\therefore A\cup \left( B-A \right)\subset A\cup B$                                             ……(3)

Next we show that $A\cup \left( B-A \right)\subset A\cup B$

Let $y\in A\cup B$

$y\in A$ or $y\in B$

($y\in A$ or $y\in B$) and ($y\in A$or $y\notin A$)

$y\in A\cup \left( B-A \right)$

$\therefore A\cup \left( B-A \right)\subset A\cup B$

Hence from (3) and (4) we obtain that$A\cup \left( B-A \right)=A\cup B$

Hence proved the statement


9. Using properties of sets, show that

i. $A\cup \left( A\cap B \right)=A$

Ans-

To show that, $A\cup \left( A\cap B \right)=A$ by using the property offsets

We know that,

$A\subset A$

$A\cap B\subset A$

$\therefore A\cup \left( A\cap B \right)\subset A$                             ……(1)

And $A\subset A\cup \left( A\cap B \right)$                         ……(2)

From (1) and (2) we get that

$A\cup \left( A\cap B \right)=A$

Hence it has been showed that $A\cup \left( A\cap B \right)=A$


ii. $A\cap \left( A\cup B \right)=A$

Ans-

To show that $A\cap \left( A\cup B \right)=A$ by using the property of sets

$A\cap \left( A\cup B \right)=\left( A\cap A \right)\cup \left( A\cap B \right)$

$=A\cup \left( A\cap B \right)$

$=A$(from (1))

Hence it has been shown that $A\cap \left( A\cup B \right)=A$


10. Show that $A\cap B=A\cap C$ need not imply $B=C$

Ans-

Given that,

$A\cap B=A\cap C$

To show that the above does not imply $B=C$

Let $A=\left\{ 0,1 \right\},B=\left\{ 0,2,3 \right\},C=\left\{ 0,4,5 \right\}$

Now, 

$A\cap B=\left\{ 0 \right\}=A\cap C$

But $B\ne C$ since $2\in B$ and not to $C$

$\therefore $It has been shown that $A\cap B=A\cap C$ does not necessarily imply $B=C$


11. Let $A$ and $B$ be sets. If $A\cap X=B\cap X=\varnothing $ and $A\cup X=B\cup X$ for some text $X$, show that $A=B$

(Hints: $A=A\cap \left( A\cup X \right),B=B\cap \left( B\cup X \right)$ and use distributive law)

Ans-

Given that,

$A\cap X=B\cap X=\varnothing $

$A\cup X=B\cup X$

To show that,

$A=B$

We know that,

$A=A\cap \left( A\cup X \right)$

$=A\cap \left( B\cup X \right)$ since $A\cup X=B\cup X$

$=\left( A\cap B \right)\cup \left( A\cap X \right)$[By distributive law]

$=\left( A\cap B \right)\cup \varnothing $ since $A\cap X=\varnothing $

$=A\cap B$                                              ……(1)

Now consider$B=B\cap \left( B\cup X \right)$

$=B\cap \left( A\cup X \right)$ since $B\cup X=A\cup X$

$=\left( B\cap A \right)\cup \left( B\cap X \right)$[By distributive law]

$=\left( B\cap A \right)\cup \varnothing $ since $B\cap X=\varnothing $

$=B\cap A$

$=A\cap B$                                               ……(2)

It is possible only when $A=B$

So from (1) and (2), we get,

$A=B$


12. Find sets $A,B$ and $C$ such that $A\cap B,B\cap C$ and $A\cap C$ are non-empty subsets and $A\cap B\cap C=\varnothing $

Ans-

Given that,

$A\cap B,B\cap C,A\cap C$ are non-empty subsets and

$A\cap B\cap C=\varnothing $

Let $A=\left\{ 0,2 \right\},B=\left\{ 1,2 \right\},C=\left\{ 2,0 \right\}$

Now, 

$A\cap B=\left\{ 1 \right\}$

$B\cap C=\left\{ 1,2 \right\}$

$A\cap C=\left\{ 0 \right\}$

And $A\cap B\cap C=\varnothing $

$\therefore $It has been showed that $A\cap B,B\cap C$ and $A\cap C$ are non-empty subsets but however $A\cap B\cap C=\varnothing $


13. In a survey of $600$ students in a school, $150$ students were found to be taking tea and $225$ taking coffee, $100$ were taking both tea and coffee. Find how many students were taking either coffee or tea?

Ans-

Let $U$ be the set of people who took part in the survey

Let $T$be the set of people who like tea

Let $C$be the set of people who like coffee

Given that,

$n\left( U \right)=600,n\left( T \right)=150,n\left( C \right)=225$

$n\left( T\cap C \right)=100$

To find,

The number of students who neither like coffee or tea$\left( n\left( T'\cap C' \right) \right)$

$n\left( T'\cap C' \right)=n\left( T\cup C \right)'$

$=n\left( U \right)-n\left( T\cup C \right)$

$=n\left( U \right)-\left[ n\left( T \right)+n\left( C \right)-n\left( T\cap C \right) \right]$

$=600-\left( 150+225-100 \right)$

$=325$

$\therefore $The number of students who like neither coffee or tea is $325$


14. In a group of students, $100$ know Hindi, $50$ Know English and $25$ know both. Each of the students knows either Hindi or English. How many students are in this group?

Ans-

Let $U$ be the set of all students in this group

Let $E$ be denote the students who know English and $H$ denote the students who know Hindi

Given that,

$n\left( H \right)=100$

$n\left( E \right)=50$

$n\left( H\cap E \right)=25$

And $H\cup E=U$

To find,

The number of students in this group

$n\left( H\cup E \right)=n\left( H \right)+n\left( E \right)-n\left( H\cap E \right)$

$U=100+50-25$

$=125$

$\therefore $The total number of students in the group is $125$.


15. In a survey of $60$ people, it was found that $25$ people read newspaper $H$, $26$ read newspaper $T$, $26$read newspaper $I$, $9$ read both $H$ and $I,11$ read both $H$ and $T,8$ read both $T$ and $I,3$ read all three newspapers. Find:

i. The number of people who read at least one of the newspapers

Ans-

Let $A,B,C$ denote the people who read the newspaper $H,T,I$ respectively

Given that,

$n\left( A \right)=25$

$n\left( B \right)=26$

$n\left( C \right)=26$

$n\left( A\cap C \right)=9$

$n\left( A\cap B \right)=11$

$n\left( B\cap C \right)=8$

$n\left( A\cap B\cap C \right)=3$

To find the number of people who read at least one of the newspapers

$n\left( A\cup B\cup C \right)=n\left( A \right)+n\left( B \right)+n\left( C \right)-n\left( A\cap B \right)-n\left( B\cap C \right)-n\left( C\cap A \right)+n\left( A\cap B\cap C \right)$

$=25+26+26-11-8-9+3$

$=52$

$\therefore 52$ people read at least one of the newspapers


ii. The number of people who read exactly one newspaper

Ans-

Let $A,B,C$ denote the people who read the newspaper $H,T,I$ respectively

Given that,

$n\left( A \right)=25$

$n\left( B \right)=26$

$n\left( C \right)=26$

$n\left( A\cap C \right)=9$

$n\left( A\cap B \right)=11$

$n\left( B\cap C \right)=8$

$n\left( A\cap B\cap C \right)=3$

To find,

The number of people who read exactly one newspaper

Let $a$ be the number of people who read newspapers $H$ and $T$ only

Let $b$ be the number of people who read newspapers $H$ and $I$ only

Let $c$ be the number of people who read newspapers $I$ and $T$ only

Let $d$ be the number of people who read newspapers $H,I$ and $T$ 

$n\left( A\cap C \right)=b+d=9$

$n\left( A\cap B \right)=a+d=11$

$n\left( B\cap C \right)==c+d=8$

$n\left( A\cap B\cap C \right)=d=3$

(Image Will Be Updated Soon)

$a+d+c+d+b+d=11+8+9$

$=28$

$a+b+c+d=28-2d$

$=28-6$

$=22$

Number of people read one newspaper $=52-22$

$\therefore 30$ people read exactly one newspaper


16. In a survey it was found that $21$ people like product $A$, $26$ people like product $B$, $29$ people like product $C$. If $14$ people liked products $A$ and $B$, $12$ liked products $C$ and $A$, $14$ liked product $B$ and $C$ and $8$ liked all the three products. Find how many like the product $C$ only.

Ans-

Let $A,B,C$ denote the people who like product $A,B,C$ respectively

Given that,

$n\left( A \right)=21$

$n\left( B \right)=26$

$n\left( C \right)=29$

$n\left( A\cap C \right)=12$

$n\left( A\cap B \right)=14$

$n\left( B\cap C \right)=14$

$n\left( A\cap B\cap C \right)=8$

(Image Will Be Updated Soon)

From the diagram, we can observe that,

$n\left( C \right)=29-\left( 4+8+6 \right)$

$=29-18$

$=11$

$\therefore 11$ people like only the product $C$

NCERT Solutions for Class 11 Maths Chapter 1 - Sets | Free PDF

Class 11 Maths Chapter 1 Solutions

The application of Sets lies in defining the concepts of relations and functions. Some of them are covered in this chapter. Ch 1 Maths Class 11 is categorised in different sections. Starting with the introductory part, it teaches you about Set. You can refer to the given examples to understand it better. Next, come the Sets and their representative topic which deals with the various types of sets. Here is an example of a problem from this section:

1. Write Down The Below-mentioned Sets in Set-builder Form:

a) {3, 6, 9, 12}

b) {2, 4, 8, 16, 32}

c) {5, 25, 125, 625}

d) {2, 4, 6, .....}

e) {1, 4, 9, ....., 100}

Sets Class 11 NCERT Solutions comprises Empty Sets, Finite and Infinite Sets, Sub Sets and Equal Sets. Furthermore, the Sub Set topic is divided into more sub-topics like intervals of subsets of R and a subset of a set of real numbers.

 

Examples of Questions:

1. Determine Which of The Following Sets Are Infinite or Finite

a) Set of alphabets in English

b) Set of animals on earth

c) Set of number which are multiples of 5

2. Which of The Following Sets Are Equal Sets? Find Out.

a) M = {2, 4, 8, 12}, N = {1, 2, 3, 4}, O = {4, 8, 12, 14}, P = {3, 1, 4, 2}

b) A = {-1, 1}, B = {0, a}, C = {1, -1}, D = {0, 1}

Sets Class 11 NCERT Solutions include Venn Diagram, Power Set, Universal Set and Operations. Again, the last topic is further categorised into union, difference and intersection of sets.

Question Examples:

1. What Will Be The Universal Set (S) For The Following?

a) A set of right angles

b) A set of scalene or isosceles triangle

2. Determine The Intersection of The Following Set Pairs.

a) A = {1, 3, 5}, B = {1, 2, 3}

b) X = {a, e, i, o, u}, Z = {a, b, c}

In Sets Class 11 NCERT Solutions, the final two topics of Sets are complements of a Set and real-life problems on intersection and union of sets.

Example of a Question From This Section:

1. Among a group of students, 50 of them know English, 100 of them know Hindi and 25 know both the languages and everyone knows either of the languages. How many students are there?

Together, all of these topics may seem a bit challenging at first, but you need not fret. Before proceeding with the exercises, you need to be thorough with all the concepts. As a reference, you can take help from online NCERT Solutions for Class 11 Maths Chapter 1 PDF and go through the same to know the tips and tricks to solve the sums.

Access other exercise solutions of Class 11 Maths Chapter 1- Sets

Chapter 1 - Sets all Exercises in PDF Format

Exercise 1.1

6 Questions & Solutions

Exercise 1.2

6 Questions & Solutions

Exercise 1.3

9 Questions & Solutions 

Exercise 1.4

12 Questions & Solutions

Exercise 1.5

7 Questions & Solutions 

Exercise 1.6

8 Questions & Solutions


Why Should Students Download Class 11 Maths NCERT Solutions Chapter 1?

Following are the reasons why it is recommended to download Ch 1 Maths Class 11 PDF material:

  1. Class 11 Sets solutions are written by scholars and rigorously follow the guidelines mandated by the CBSE Board.

  2. The solutions are provided concisely, and step-by-step approach makes it easier for learners to comprehend.

Sometimes, due to lack of accurate reference materials, students find it challenging to understand and solve the problems. However, by making use of Sets Class 11 NCERT PDF presented by Vedantu, learners can stay assured to receive desired scores in exams.

Make sure to download the PDF today!


Key Features of NCERT Solutions for Class 11 Maths Chapter 1: Sets

This chapter contains six exercises and a miscellaneous activity to help students understand fully the topics related to Sets in the Class 11 Maths CBSE Syllabus (2024-25). The following are the key features of NCERT Solutions for Class 11 Maths Chapter 1:

  • Properly arranged solutions in the right manner to help students in quickly finding solutions.

  • Detailed solutions to questions that are simple to grasp and learn.

  • Expertly designed to match the curriculum.

  • NCERT solutions help students develop a strong conceptual foundation, which is important in the final stages of preparation for competitive exams.

  • Students can easily prepare all of the ideas covered in their respective classes much more efficiently and even crack the most difficult competitive exams.


Conclusion

NCERT Solutions for Class 11 Maths Chapter 1 - Sets offered by Vedantu is an excellent resource for students who want to excel in their mathematical studies. The solutions are designed by experts according to the CBSE syllabus for Class 11 students and cover all the important topics in the chapter. They provide a comprehensive and detailed explanation of the concepts of sets, making it easier for students to understand and improve their mathematical skills. Additionally, the solutions offer practice exercises and questions that help students test their understanding of the chapter and prepare for their exams. Vedantu also provides interactive live classes and doubt-solving sessions to help students clarify their doubts and improve their understanding of the chapter. Overall, the NCERT Solutions for Class 11 Maths Chapter 1 - Sets offered by Vedantu are an essential resource for students who want to improve their mathematical skills and score well in their exams.

FAQs on NCERT Solutions for Class 11 Maths Chapter 1 - Sets

1. What are the applications of Sets?

There are several applications of Sets in both mathematics and real life. Starting from formulating Calculus, Geometry and Topology to the formation of Algebra around rings, fields and groups. Plus, it also has various uses in fields like Chemistry, Physics, Biology, Computer Science and Electrical Engineering.

2. How many problems are there in each exercise of NCERT Solutions of Maths Class 11 Chapter 1?

Overall, there are six exercises along with a miscellaneous one. The number of questions for Ex. 1.1 is 6, Ex. 1.2 is 6, Ex. 1.3 is 9, Ex. 1.4 is 12, Ex. 1.5 is 7 and Ex. 1.6 is 8. Plus, the last exercise contains sixteen questions. You can easily get solutions to the problems in the form of PDF on web portals. Make sure to download them and refer to while preparing for your exam.

3. What topics are included in Class 11 Maths Chapter 1 Sets?

Class 11 Chapter 1 is Sets. It is generally referred to as concepts of relations and functions. It includes some of the fundamental operations and definitions that involve sets. This topic is counted as one of the simplest ones in the curriculum of Class 11, and students can score full marks in this section. With regards to the chapter topics, they comprise of Introduction, Sets and their Representations, The Empty Set, Infinite and Finite Sets, Equal Sets, Subsets, Power Sets, Universal Set, Venn Diagrams, Union of Sets, Operation on Sets, Intersection of Sets, Complement of a Set and practical problems on intersection and union of 2 separate Sets.

4. What are the basics of Class 11 Maths Chapter 1 - Sets?

Sets are used for defining functions and relations. The concept of Sets is covered in Chapter 1 of the Math NCERT textbook. This is considered an easy chapter, and you can score maximum marks in your exam. There are some basic definitions of the types of sets and a few formulas that you need to learn to become well-versed in this chapter. In Vedantu’s NCERT Solutions for Class 11 Maths, you will learn a way of solving these questions in an easy and quick way.

5. What are the real-time applications of Class 11 Maths Chapter 1 - Sets?

Sets are the foundation for several complex concepts. From the formulation of logical foundations for calculus, topology, and geometry to algebra related to the field, groups, and rings, Sets have a lot of applications in several fields of science and mathematics. This includes fields like physics, chemistry, biology, and electrical and computer engineering. You need to have a strong understanding of sets. For this, you can refer to Vedantu's NCERT Solutions for Class 11 Maths Chapter 1 - Sets.

6. What are the most important definitions that come in Class 11 Maths Chapter 1 - Sets?

Below are some of the most important definitions that you will be learning in Class 11 Maths Chapter 1 - Sets:

  1. Empty Sets - An empty set is a set that has null elements or no elements.

  2. Singleton Set - A set, which has one element is called a Singleton set.

  3. Finite Set - A set with a finite number of elements is called a finite set.

  4. Infinite Set - A set with an infinite number of elements is called an infinite set.

  5. Equal Set - Two sets are said to be equal when every element of one set is also an element of the other set and vice versa.

7. What are the most important theorems that come in Class 11 Maths Chapter 1 - Sets?

Here are some important theorems that you will learn in Class 11 Maths Chapter 1 - Sets:

  • The union of two sets, A and B contain elements that are in set A and set B.

  • The intersection of sets A and B contain common elements from set A and B.

  • The complement of set A is the set of elements from the universal set U that are not in A.

These theorems will help you solve the questions related to Sets.

8. What are the most important formulas that I need to remember in Class 11 Maths Chapter 1 - Sets?

Here are some important formulas you need to remember from Class 11 Maths Chapter 1 - Sets:

  1. If (A∪B)=ϕ, then n(A∪B)=n(A)+n(B)−n(A∩B)

  2. If (A∩B)=ϕ, then n(A∪B)=n(A)+n(B)

  3. (A∩B)′=A′∪B′

  4. (A∪B)′=A′∩B′

You can find the solutions to these in Vedantu’s NCERT Solutions for Class 11 Maths Chapter 1- Sets on Vedantu (vedantu.com) free of cost. You can download the solutions from the Vedantu mobile app as well.